87
JMET 2006 Original Paper SECTION 1: VERBAL COMMUNICATION (Questions 1 – 40) Directions: Questions 1 – 3 relate to the passage given below: Number of words in this passage : 325 When I started working during the late 1960s and early ’70s I was the proud owner of a slide rule. It was a low-mileage model, as I only knew how to work out percentages on it, but even that was better than struggling with long multiplication or logarithms to do the same work as some of my colleagues were wont to do. The point is that this was only three decades ago, and the pocket calculator had still not been invented. I remember, in the early 1970s, sitting in a meeting in the viewing room of the advertising agency I worked for, taking part in a discussion with our client, Procter and Gamble, on whether the commercial which had just been approved should be shot in black-and-white, or colour. The discussion, as with most discussions with that client, was long and carefully articulated on both sides. The agency, of course, looked to the future, and argued strongly for colour. We were finally overruled, on the grounds that there were still too few colour TV sets in existence for it to be worth the extra investment in colour film. At the same period, I recall the excitement of the company’s first computer being delivered. The accounts’ office window was temporarily removed, while the computer was swung into place by a crane especially hired for the purpose. The computer power was probably less than a Personal Organiser. Not only was colour television a rarity, and the personal computer still some way off, other everyday objects had still to be invented, like the digital watch or the camcorder. How we existed without such basic everyday tools I now find hard to imagine. The truth is that we and our parents and grandparents before us – and their forebears before them stretching back over the past two centuries – have seen and accommodated huge technical advances and social changes. Many of these changes have not only been big – they’ve been fast. 1. Which of the following inferences in the MOST APPROPRIATE as per the passage above? (A) Change is a process of struggling against existing ideas. (B) The world has changed rapidly since the 1960s and ‘70s. (C) Change is exciting, especially in the field of technology. (D) The world has been changing significantly for a long time.

JMET 2006 Original Paper

Embed Size (px)

Citation preview

Page 1: JMET 2006 Original Paper

JMET 2006 Original Paper 

SECTION 1: VERBAL COMMUNICATION (Questions 1 – 40) Directions: Questions 1 – 3 relate to the passage given below: Number of words in this passage : 325 

When I started working during the late 1960s and early ’70s I was the proud owner of a slide rule. It was a low-mileage model, as I only knew how to work out percentages on it, but even that was better than struggling with long multiplication or logarithms to do the same work as some of my colleagues were wont to do. The point is that this was only three decades ago, and the pocket calculator had still not been invented. I remember, in the early 1970s, sitting in a meeting in the viewing room of the advertising agency I worked for, taking part in a discussion with our client, Procter and Gamble, on whether the commercial which had just been approved should be shot in black-and-white, or colour. The discussion, as with most discussions with that client, was long and carefully articulated on both sides. The agency, of course, looked to the future, and argued strongly for colour. We were finally overruled, on the grounds that there were still too few colour TV sets in existence for it to be worth the extra investment in colour film. At the same period, I recall the excitement of the company’s first computer being delivered. The accounts’ office window was temporarily removed, while the computer was swung into place by a crane especially hired for the purpose. The computer power was probably less than a Personal Organiser. Not only was colour television a rarity, and the personal computer still some way off, other everyday objects had still to be invented, like the digital watch or the camcorder. How we existed without such basic everyday tools I now find hard to imagine. The truth is that we and our parents and grandparents before us – and their forebears before them stretching back over the past two centuries – have seen and accommodated huge technical advances and social changes. Many of these changes have not only been big – they’ve been fast. 1.   Which of the following inferences in the MOST APPROPRIATE as per the passage above?

(A) Change is a process of struggling against existing ideas.(B) The world has changed rapidly since the 1960s and ‘70s.(C) Change is exciting, especially in the field of technology.(D) The world has been changing significantly for a long time.

 Explanatory Note:The concluding paragraph clearly states that changes have been taking place over the past two centuries. Choice (D) is the most appropriate.                                                                                                                                           Choice (D)

 2.   Which of the following statements is MOST ACCURATE in terms of the passage above?

(A) Colour commercial was ruled out because colour photography was in its infancy.(B) An office window had to be dismantled so as to put the computer inside.(C) The digital camcorder had not yet been invented in the 1970s.(D) Slide rules could calculate percentages, multiplication and logarithms

 Explanatory Note:Choice (C) is the most appropriate answer. The passage says colour TV sets were few – it does not say colour photography was in its infancy. Paragraph 4 says that the office window was

Page 2: JMET 2006 Original Paper

temporarily removed. Hence choice (B) is also wrong. From the first paragraph we can infer that choice (D) is also not accurate.  Choice (C)

 3.   The MOST APPROPRIATE title for the above passage could be

(A) “Changes since the 1960s”     (B) “Accepting change”           (C) “Changes and obsolescence”  (D) “The changing world”

 Explanatory Note:The passage, in general, talks about the changes which have taken place in the world over the past two centuries. Hence the title ‘the changing world’ seems to be the most appropriate for the passage.                                                        Choice (D)

Directions: In Questions 4 and 5, choose the option which is OPPOSITE in meaning to the CAPITALIZED word. 4.   TYRO

(A) Expert                              (B) Tyrannical                         (C) Beginner                          (D) Tycoon

 Solution:The word ‘expert’ is an antonym of the word ‘tyro’ (an inexperienced person).                                                        Choice (A)

5.   STIGMA(A) Horror                              (B) Humour                            (C) Honour                             (D) Holler

 Solution:‘Honour’ is an antonym of ‘stigma’ (dishonour).                                                                                                  Choice (C)

Directions: In Questions 6 and 7, fill in the blanks with the option containing the MOST APPROPRIATE set of words. 6.   We are more likely to believe a story told by someone we consider _______ than someone we

have learnt not to trust, so the reputation of the narrator ______ the story.(A) spurious, negates             (B) honest, validates              (C) objective, officiates                 (D) adorable, dominates

 Solution:The sentence states that we do not believe a story which is told by a person whom we do not trust Therefore we are more likely to believe a story told by an honest person. Thus the reputation of the narrator ‘validates’ (authenticates) the story. The words ‘spurious’ (false) and ‘negates’, ‘objective’ (unbiased) and ‘officiates’ (supervises) ‘adorable’ and ‘dominates’ are not apt in the given context.                                                                                                                                                    Choice (B)

 .   This will require a culture of _______ and should allow diversity to _______.

(A) sympathy, display             (B) stamina, flaunt                  (C) resilience, brandish           (D) tolerance, flourish

 Solution:Logically, option D is the most appropriate because only a culture which is tolerant will allow diversity to flourish. Although the first words of options A and C may be possible in the first blank, the words ‘display’ and ‘brandish’ (flaunt) are inappropriate in the second blank.                                                                                                                                            Choice (D)

Directions: In Questions 8 and 9, carefully read the sentences given below and identify the grammatically INCORRECT option to fill in the blank. 8.   There are more flowers here _______ earlier.

(A) than there used to be             (B) than there was                       (C) than there had been               (D) than there were

Page 3: JMET 2006 Original Paper

 Solution:Option B is incorrect because ‘flowers’ is plural. Hence it should be denoted by the plural verb ‘were’.                     Choice (B)

9.   This strategy has proved very successful for Nike, but to keep ahead of competition they must keep _______ new factory sites and sourcing cheap workers.(A) searching                         (B) exploring                          (C) surveying                         (D) scrutinizing

 Solution:The word ‘searching’ is the only incorrect option, for ‘searching’ always takes the preposition ‘for’.                         Choice (A)

Directions: In Questions 10 and 11, choose the option that is CLOSEST in meaning to the CAPITALIZED word. 10. GRIT

(A) Grid                            (B) Grind                               (C) Grin                                       (D) Grill 

Solution:The word ‘grind’ is closest in meaning to the word ‘grit’ because the word grit means ‘clench’ (teeth).                     Choice (B)

1. INEXPLICABLE(A) Incomprehensible             (B) Indelible                           (C) Inextricable                            (D) Infallible

 Solution:The word ‘inexplicable’ means ‘incomprehensible’.                                                                                              Choice (A)

Directions: In Questions 12 – 14, select the pair of words that best expresses a relationship SIMILAR to the pair in CAPITAL letters. 12. COMMUNE : COMMUTE

(A) Converse : Interact            (B) Announce : Lighten           (C) Communicate : Travel             (D) Declare : Exchange

 Solution:Option C seems to be the most appropriate answer because the words ‘commune’ and ‘communicate’ are synonymous. Similarly the words ‘commute’ and ‘travel’ convey the same meaning.                                                                  Choice (C)

13. PIRATE : PIROUETTE(A) Marauder : Dive           (B) Criminal : Tumble        (C) Convict : Spin             (D) Buccaneer : Gyrate

 Solution:Only option D best expresses the relationship stated in the capitalized pair because the words ‘pirate’ and ‘buccaneer’ are synonyms. Similarly ‘pirouette’ and ‘gyrate’ are synonymous.                                                                             Choice (D)

14. PROVINCIALISM : PAROCHIALISM(A) Dilettante : Inexpert          (B) Miscegenation : Mitigation      (C) Meretricious : Sober         (D) Decadent : Resplendent

 Solution:The words ‘provincialism’ and ‘parochialism’ express a synonymous relationship. Similarly the words ‘dilettante’ and ‘inexpert’ are synonymous because a dilettante pursues a particular art only as a hobby but has not acquired expertise in it. Therefore option A is more logical.                                                                                                                                 Choice (A)c

Directions: Question 15 consists of four groups of jumbled phrases, only ONE of which is grammatically correct. Identify the CORRECT option. 

Page 4: JMET 2006 Original Paper

15. (A) matured, venture capital has undergone still another redefinition, returning / at least in part to its early roots in funding       mature / now as the technology business has / businesses, in technology and in other segments as well(B) energy and financial services / areas and has been a leading in information technology,

media / the firm has invested across many industry / and communications, telecommunications, healthcare,

(C) in the area of new ventures that, I might one day return to help put / I left the Gaeltacht convinced that the way to / in place the very infrastructures needed to create a sustenable and thriving economy / save this island of Irish tradition is to work

(D) the consequence of that - in part – is / companies with good people that are just farther along / that have gone public in the past that are still good, / that have gone public in the past that are still good, / that there are a lot of orphans out three among companies

 Solution:The sentences are as follows:(A) Now as the technology business has matured, venture capital has undergone still another

redefinition, returning at least in part to its early roots in funding mature businesses, in technology and in other segments as well.

(B) The firm has invested across many industry areas and has been a leading in information technology, media and communications, telecommunications, healthcare, energy and financial services.

(C) I left the Gaeltacht convinced that the way to save this island of Irish tradition is to work in the area of new ventures that, I might one day return to help put in place the very infrastructures needed to create a sustenable and thriving economy.

(D) In choice (D), the given phrases do not form a meaningful sentence.In choice (B), the sentence should read “… and has been a leader…”, not ‘leading’. In choice (C), it should read ”… new ventures so that …”. The meaning conveyed here is that the work in the area of new ventures has to be in such a way that …. In such a context, “so … that” would be appropriate. Only choice (A) is grammatically correct.                              Choice (A)

Directions: The direct speech in Question 16 has been rewritten as reported speech (indirect form) in the four options given below. Identify the grammatically CORRECT option. 16. He said, “I need to be there in person or my budget will be reduced. However, I will attend the meeting before I leave.”

(A) He said that he needed to be there in person or his budget will be reduced. But he will attend the meeting before he left.(B) He said, he needed to be there in person or his budget will be reduced. However, he’ll attend the meeting before he leaves.(C) He said that he needed to be there in person or his budget would be reduced. However, he

said that he would attend the meeting before he left.(D) He said he needs to be there in person, or his budget would be reduced. But he will attend the meeting before he left.

 Solution:Option B is incorrect because ‘that’ is missing in the reported speech. Option A is incorrect because ‘will’ should change to ‘would’ in the reported speech. D is incorrect because the verb ‘needs’ should be in the past tense in the report speech. Only option C is correct.                                                                                                                                         Choice (C)

Directions: Questions 17 – 19 refer to the passage below: Number of words in this passage : 211 

To make effective decision, all we have to do is to out-think our opponent. Our decision needs to be better than his, that’s all. There is no need to be perfect. The mistake we make is to think through our intellect. If we ask any successful business leader or CEO these days about what has made them so successful, again and again they claim that their

Page 5: JMET 2006 Original Paper

success came from something beyond their intellect; something beyond logic and facts; something that gave them the intelligence and guts to take effective decisions. It is simply the intuitive power within them that has helped them make these effective decisions. We can tune into intuition as part of our regular life. The question therefore is not whether intuition is an inborn quality, but whether we can make ourselves intuitive. When we step into the present, we step out of time-bound awareness is mass, which is solid. Non time-bound awareness is pure energy, liquid, dynamic, bubbling and creative. We step out of our boundaries. We become free. We become intuitive. When our thoughts stop, our present vision extends into the past and the future. We become free of time and space constraints. When we meditate deeply, we become intuitive, and can reach cosmic intelligence or enlightenment. 

17. According to the passage, a successful business leader(A) relies mostly on logic and facts.(B) is more intelligent than his / her opponents.(C) takes effective decisions.(D) is free of time and space constraints.

 Explanatory Note:Choice C is the most appropriate choice. The first paragraph states that the success of leaders comes from something beyond logic and facts. Nowhere is it stated that a leader is more intelligent than his or her opponents. The same paragraph also states that a leader takes effective decisions. When one’s vision extends into the past and future, one becomes free of time and space constraints. This can happen to anybody and not just a successful leader.                                                          Choice (C)

 18. The MOST APPROPRIATE title for the passage might be

(A) “Beyond intuitive boundaries”                               (B) “Intuition, the key to effective decisions”(C) “Intuitive leadership”                                            (D) “Intuitive reasoning”

 Explanatory Note:The entire passage focuses on how intuition helps in making effective decisions. That leaders have good intuition is merely another point stated to emphasize that intuition helps one make effective decisions and therefore be successful. It does not focus on intuitive leadership or reasoning. Choice B would be the most appropriate title.                                        Choice (B)

 19. Which of the following statements CANNOT be directly inferred from the above passage?

(A) Effective decisions are not necessarily perfect decisions(B) successful CEOs have intuitive powers(C) To be intuitive, we have to step out of our time-boundaries(D) Meditation helps us take effective decisions

 Explanatory Note:Choice A can be directly inferred, for the paragraph opens by saying that effective decisions need not be perfect. From the second paragraph we can infer that successful CEOs have inituitive powers.  Choice C can be inferred from paragraph 4. Choice D is the most appropriate answer.                                                                                                                    Choice (D)

Directions: In Question 20, ONE of the options is grammatically INCORRECT. Identify the INCORRECT option. 20. (A) They speak French in parts of Italy as well as France

(B) It looks as though John is going to change his job(C) Smoking is dangerous as well as it makes you smell bad(D) As well as birds, some mammals can fly

 Solution:

Page 6: JMET 2006 Original Paper

Option (C) is grammatically incorrect. The sentence should read, “Smoking is dangerous and it also makes you smell bad”.                                                                                                                                                                        Choice (C)

Directions: In Questions 21 – 23, identify the grammatically CORRECT option. 21. (A) If you didn’t study English at school, you won’t understand this paper

(B) If you don’t study English at school, you wouldn’t understand this paper(C) If you didn’t study English at school, you wouldn’t understand this paper(D) If you haven’t studied English at school, you wouldn’t understand this paper

 Solution:Only option (C) is correct. In a sentence with an ‘if’ clause, if the simple past is used in the ‘if’ - clause then the main clause takes ‘would’ or ‘wouldn’t’. Only option (C) follows this format.                                                                             Choice (C)

22. (A) Music – Classical music, was one of her strong points          (B) Beside art, she also loved Chamber music

(C) The romantic number, in spite of the rain, was still audible     (D) The room was empty except the music album

 Solution:Only sentence C is right. Sentence A is incorrect as ‘classical music’ is separated from the sentence with a dash on one side and a comma on the other. It should have a dash on both sides or a comma on both sides. B should read “Besides art”, …. ‘Beside’ means ‘next to’ and is inappropriate in this context. ‘Besides’, which means ‘in addition to’, is more appropriate. In sentence D ‘except’ should be followed by ‘for’.                                                                                                 Choice (C)

23. (A) The train may be late, as it happened yesterday                        (B) Some people are interested, but the majority doesn’t care(C) My father, whom we hope will be out of hospital soon, will come(D) She works hardest when she’s working for her family

 Solution:Only Choice (A) is grammatically correct. ‘Majority’, (in B), takes a plural verb – it should be ‘do not’. In choice (C), ‘whom’ should be ‘who’. Moreover, the phrasing is incorrect. It should read “who we hope will soon be out of hospital.” Choice (D) is erroneous because the superlative is always preceded by ‘the’.                                                                           Choice (A)

Directions: Questions 24 – 26 relate to the following article: Number of words in this passage : 388 

Small and Medium Enterprises (SMEs) played a crucial role in the development of India during the past 50 years. This sector constitutes about 95% of industrial units, and about 40% of total industrial output. Its direct and indirect exports potential stand at about 38%. With about 3.6 million SSI (Small Scale Industries) registered units employing close to two crore people, its employment potential is next only to the agriculture sector. Thus the performance of SMEs is important for the economic and social development of the country. One of the ways by which this sector can be made to grow fast is tapping both domestic as well as international markets through business linkages between Multinational Corporations (MNCs) and SMEs. Many OECD (Organisation for Economic Cooperation and Development) countries and some Asian ones have specific policies for developing business linkages between SMEs and MNCs. So, the SME sector in these countries have witnessed favourable growth and helped boost their countries’ exports in a very competitive way. For example, Thailand ensured that the state provided industry with physical infrastructure and technological resources. Other Asian governments (Republic of Korea, Malaysia, Province of Taiwan, China) have included various incentives in the form of tax breaks, preference in public contracts and soft credit lines for both Transnational Corporations (TNCs) and SMEs to intensify relations and technology transfer.India, however, has no specific policy

Page 7: JMET 2006 Original Paper

guidelines to develop linkages between SMEs and MNCs. Consequently, the vast majority of SMEs that cannot meet the requirements set by MNCs remain totally de-linked. In business linkages between SMEs and MNCs, outsourcing and value chain management started to play a key role, since MNCs could not become competitive without a competitive supplier base. Thus, to create a capable supplier base, a three-pronged approach to developing research, development and innovation in SMEs by accelerating their rate of technology acquisition through realization of R & D projects. The government must promote and strengthen the knowledge base and competencies in SMEs in terms of quality, productivity and cost. Hence, there is an urgent need to strengthen the National Entrepreneurship Development Board (NEDB) and formulate a suitable plan for promoting rural entrepreneurship. Substantial investments and physical infrastructure development are needed all over the country. In this regard, a public-private partnership needs to be developed for implementing and reviewing the supportive measures at frequent intervals. 

24. According to the article, the Indian government can help facilitate the SME-MNC linkage by(A) selecting potential local firms as suppliers to MNCs(B) providing technological resources through investment in R & D(C) providing soft credit lines for MNCs(D) training SMEs in outsourcing and value chain management

 Explanatory Note:The last paragraph clearly says that there is a need to support development in SMEs through investments in R and D projects.                                                                                                                                                                     Choice (B)

 25. The article DOES NOT talk about

(A) role of business associations in fostering SME-MNC linkages(B) role of developing countries in fostering SME-MNC linkages(C) role of the government in fostering SME-MNC linkages(D) role of MNCs in developing the SME sector Explanatory Note:Choice (A) is the most appropriate answer. The article talks about the role of developing countries, the intuitive government and the MNCs – but it does not mention that business associations are necessary to foster SME – MNC linkages.      Choice (A)

 26. As per the passage, only ONE of the following statements is correct. Identify the correct statement.

(A) The government should take the lead in facilitating SME-MNC linkages(B) The only way to facilitate growth of SME sector is through business linkages with MNCs(C) The government should help SMEs in improving their knowledgebase and competencies(D) SMEs currently export 38% of total industrial output

 Explanatory Note:Choice (C) is correct. Option D is not the answer as the passage does not mention that the government should take the lead in facilitating SME-MNC linkage. Business linkages with MNCs is one of the ways of facilitating the growth of SME sector but not the only way. Hence B is ruled out.                                                                                                                  Choice (C)

Directions: Read the paragraph below and answer Questions 27 – 29 which follow: Number of words in this passage : 51 

Confusion is internal and/or external chaos. Faulty implications, cognitive distortions, interpersonal disruptions, and complex sign of confusion and conflict promote a spirit or atmosphere of misinformation, misinterpretation and miscommunication. Where there is considerable friction in the mix of expressive freedom and interpretative response, an atmosphere of uncertainty and commotion will prevail.

Page 8: JMET 2006 Original Paper

 

27. Which of the following, according to you, is the MOST APPROPRIATE title for the passage?(A) “Causes of uncertainty”                                 (B) “Misinterpretation and commotion”(C) “Defining confusion”                                      (D) “Uncertainty – the cause of commotion”

 Explanatory Note:The main focus of the passage is on confusion. Therefore C would be the most appropriate title. The others are merely subsets of the main idea.                                                                                                                                            Choice (C)

 28. The MOST APPROPRIATE meaning of “commotion,” as used in the passage, would be

(A) a state of agitation and disturbance               (B) a mental condition that leads to emotional breakdown(C) a state of uncertainty leading to depression    (D) a condition of confusion and faulty implication

 Explanatory Note:Choice (A) is the most appropriate answer “Commotion” as used in the passage means a state of agitation and disturbance.                                                                                                                                                                       Choice (A)

 29. The passage implies that

(A) misinterpretation and miscommunication are promoted to some extent because of confusion(B) cognitive distortion and interpersonal disruption cause confusion(C) confusion is a result of misinformation, miscommunication and non-expressive freedom(D) considerable friction and cognitive distortion necessarily lead to commotion

 Explanatory Note:Choice (A) is the most appropriate answer. Cognitive distortion and interpersonal description cause misinformation and miscommunication – not confusion. Confusion promotes misinformation and is not a result of it. Cognitive distortion does not lead to commotion.                                                                                                                                         Choice (A)

Directions: Read the passage carefully and answer the Questions 30 – 33 which follow. Number of words in this passage : 323 

Although broad generalizations always oversimplify complex realities, we find numerous truths in the contrast between hierarchical, industrial manufacturing firms that dominated most of the twentieth century and today’s service-based and knowledge-sensitive organizations. When industry meant repeatedly carrying out standard, well-defined tasks and workers were seen metaphorically (and sometimes literally) as parts of a machine, progress could still be made when the social networks and relationships of individual employees were ignored or discouraged. In fact, those firms strongly depended on social capital and sometimes suffered from lack of it. Without some level of trust, respect, and generalized reciprocity, coordinated work of any kind is hard to accomplish. Still, as Henry Ford has commented, a certain rough logic lies behind treating people like cogs in a machine when you only expect and want them to do machine-like work. But very little of the work of today’s knowledge firm is repetitive or mechanical. It requires responsiveness, inventiveness, collaboration and attention. Judgement, persuasiveness, shared decisions, the pooling of knowledge, and the creative sparks people strike off one another depend on engagement with the work and one another, on the commitment that makes one genuinely a member of an organization rather than simply an “employee” - the user and the used - those terms really belong to the industrial-age model and are inappropriate to the kind of work and working relationships we consider here. Today’s most economically productive work is largely voluntary, in the sense that doing it well calls for a willing engagement of the whole self in the task. “Going through the motions” is

Page 9: JMET 2006 Original Paper

insufficient when the motions are not prescribed but change as you go along. In our view, the firm is neither a machine with each cog firmly in place performing its clearly defined task nor an unorganized (or self-organising) flock of opportunistic entrepreneurs pursuing their individual destinies. It is – it should be – a social organization of people willingly engaged in a joint enterprise. 30. In the light of your reading of the passage above identify the option that contains the set of

words CLOSEST in meaning to the set of words in CAPTIALlettersMETAPHORICALLY : STANDARD : COG : ENGAGEMENT(A) Allegorically : Bench-mark : Small Part : Obligation               (B) Figuratively : Average : Small wheel : Appointment(C) Illustratively : Criterion : Small teeth : Undertaking                 (D) Symbolically : Routine : Small Component : Commitment

 Explanatory Note:The passage says “… repeatedly carrying out standard, well-defined tasks …” (para 1, line 4). In this context, ‘standard’ could only be ‘routine’. Similarly, the second paragraph says that for productive work, a willing engagement of the whole self is necessary. In this context, the best substitute for ‘engagement’ is ‘commitment’. Hence option (D) is most appropriate.                                                                                                                                                                               Choice (D)

 31. Based on your reading of the above passage, identify the INCORRECT option.

(A) A knowledge firm requires participation and collaboration       (B) A knowledge firm encourages trust and reciprocity(C) A knowledge firm does not have employees                          (D) A knowledge firm has flexible work descriptions

 Explanatory Note:The passage only states that rather than calling a person an ‘employee’, he should be called a ‘member’ of the organization. The difference is only in the term – the employees are called members. However, we cannot say that a knowledge firm does not have employees at all.                                                                                                                                    Choice (C)

 32. The above passage implies that

(A) service-based, knowledge-intensive organizations dominate in today’s business scenario(B) employees carry out standard, well-defined tasks(C) a voluntary worker is likely to be an engaged worker(D) industrial firms cannot be economically productive Explanatory Note:The second paragraph states that today’s most economically productive work is largely voluntary. Thus we infer that a voluntary worker is likely to be an engaged worker.                                                                                                              Choice (C) 

33. The central idea of the passage relates to(A) the key difference between industrial and knowledge-intensive firms(B) the key difference between an “employee” and a “member” in an organization(C) “Mechanical” vs. “Voluntary” organizations(D) importance of social capital in creating engaged organizations

 Explanatory Note:The whole passage talks about the difference between industrial firms and knowledge firms. The difference between an ‘employee’ and a ‘member’ is one of the points – it is not the main idea. The question of social capital does not arise.                                                                                                                                                                                  Choice (A)

Directions for questions 34 – 37 relate to the passage given below: Number of words in this passage : 307 

Page 10: JMET 2006 Original Paper

Visualisation is the three-dimensional, multicoloured, singing-and-dancing version of affirmations that enables the subconscious to prefigure future achievement or success. It is a basic and fundamental human attribute, and one that can literally be the difference between surviving and not surviving. When Victor Frankl, the Freudian psychologist, was examining the discriminating factors that enabled him, and many like him, to survive in the hell of the Nazi concentration camps, the key factor was the ability to visualize. All survivors had a vision of something beyond their current suffering, something more worthwhile, and something worth hanging on for. This underlines the importance of each individual having a vision of something, outside and larger than herself, that gives her life some meaning. The very existence of a mission lifts the eyes to something more meaningful and enduring – and in so doing provides something to life for – at times when quiet surrender could be an attractive option. Such vision gives a further raison d’etre for integrity, by providing a purpose that binds together the core values that make up self-worth. One of the most powerful – and difficult to achieve – applications of visualization is to focus your mind daily on the person you intend to become. Create a clear mental picture of that person – and see it in full colour, and add sounds and smells, if they are appropriate. The emotional values you add to the visualization are vital in making the full connection to your subconscious, which acts only on thoughts that are mixed with emotions. These techniques are, of course, widely validated in fields like sport and business, where the peak performers are nearly all visualisers. There techniques are, of course, widely validated in fields like sport and business, where the peak performers are nearly all visualisers. They all see, feel, and fully experience their success before they achieve it. 34. Which of the following observations CANNOT be directly inferred from the above passage?

(A) Concentration camp survivors visualized beyond their current suffering(B) Emotional values help one’s subconscious to connect to visualization(C) Visulisation involves the ability to focus on the person you want to become(D) The existence of a goal beyond one’s current situation gives one a sense of purpose.

 Explanatory Note:Option A can be inferred from the second paragraph. According to the 3rd sentence of the last paragraph option B is true. According to the second sentence of para 3 option D is correct. Option C is a generalization whereas the passage says ‘visualisation is one of the …’.                                                                                                                           Choice (C)

 35. The above passage DOES NOT deal with

(A) uses of visulalisation         (B) quality of visualization       (C) techniques of visualization      (D) illustrations of visualization

 Explanatory Note:Choice (D) is the most appropriate answer. The first statement talks about the use of visualisation. The last paragraph talks about the quality and techniques of visualisation. But the passage does not mention any illustrations of visualisation.                                                                                                                                                                                  Choice (D)

 36. “Raison d’etre” as it is used in the above passage means

(A) the most important need                                               (B) the most important inference(C) the most important consequence                                   (D) the most important reason

 Explanatory Note:The meaning of the phrase “Raison d’etre”, as used in the passage, is the most important reason.                         Choice (D)

 37. Which of the following statements, in the light of the above passage, is NOT correct?

(A) Visualisation is the affirmation of the subconscious(B) Visualisation is three dimensional, multicoloured and auditory(C) Visualisation is a basic and fundamental quality of human mind

Page 11: JMET 2006 Original Paper

(D) Visualisation can make a significant difference in terms of our very existence 

Explanatory Note:The first and second sentences of the first para state that B and C are true. The first sentence of the third paragraph in particular and the entire passage in general state option D is correct. Option A is not stated in the passage.                           Choice (A)

Directions: In Question 38 – 40, select the set of words that best expresses a relationship SIMILAR to the set in CAPITALS. 38. SKY : BIRD : THUNDER

(A) Movie : Theatre : Actor                                            (B) Night : Stars : Music(C) Office : Conversation : Employees                            (D) Paper : Text : Watermark

 Solution:Bird and thunder are found in the sky. Similarly, text and watermark are found on paper. In choice (A), theatre is not found in a movie – it is the other way round. In choice (B), music is not found in the night. In choice (C), it is not always necessary that conversation is found in an office.                                                                                                                     Choice (D)

39. TV : VISION : MOVIES(A) Mirror : Reflection : Eye    (B) Air Conditioner : Feel : Air       (C) Oven : Food : Smell    (D) Perfume : Fragrance : Flower

 Solution:A person needs vision to see TV and a TV dishes out movies. To watch TV we need vision. Similarly, an air conditioner uses the power of feeling and gives out cool air. We need feelings to experience the cool air. In choices (A) and (C), the order is not the same. Choice (D) can be ruled out for we need not have fragrance to detect a perfume.                                   Choice (B

40. PRINTER : INK : PAPER(A) Electric Power : Desktop : Monitor                (B) Vending Machine : Water : Ground Coffee(C) Car : Brake Oil : Petrol                                 (D) Tree : Clear Polish : Timber Wood Solution:A printer uses ink on paper. Similarly a vending machine uses water on ground coffee. (A) and (D) do not have such a relationship. In Choice (C), brake oil and petrol are two distinct features – a car does not use brake oil on petrol.    Choice (B)

Directions for Questions 41 and 42: Read the given passage and answer the questions: Nine shops in a multiplex are connected by corridors. Anyone visiting these shops must begin at the reception which is part of shop number 1. From there, the other shops may be visited via different corridors connecting them. The details of the corridors connecting various shops are as under:(i)   Corridors connect the reception to shops, 2, 3 and 6.(ii)  A corridor connects the reception to shop 2 and then to shop 4.(iii) Corridors connect shops 2 and 3 to shop 7.(iv)  Shop 6 is connected to shops 3, 5 and 9.(v)  Shop 7 is connected to shop 9.(vi)  Shops 7 and 9 are connected to shop 8. 41. Without visiting any shop more than once, what is the MAXIMUM number of shops a person can visit?

(A) 5                                (B) 6                                     (C) 7                                      (D) 8 

Solution: From the given information, we can obtain the following diagram.

Page 12: JMET 2006 Original Paper

       The maximum number of shops that can be visited is 8, as in the following case.

                                                                                                                                                                           Choice (D) 42. If a visitor wants to travel to shops 4 and 5, which one of the following must be FALSE?

(A) The visitor will go to every shop except shop 3           (B) The visitor will go to shop 2 only once(C) The visitor will not visit any shop more than once       (D) The visitor will visit at least six different shops

 Solution: From the given information, we can obtain the following diagram.

      

 He has to visit shop 4 and shop 5 means he has to visit one among shop 2 and shop 6 twice.Hence, statement (C) is false                                                                                                                            Choice (C)

43. The Chinese are shorter than the Italians; the Americans are taller than the French; the French are taller than the Chinese.

From the information given above, which one of the following statement can be logically inferred?(A) The Italians are shorter than the Chinese                   (B) The French are taller than the Americans(C) The Chinese are taller than the French                      (D) The Americans are taller than the Chinese Solution:Given,Chinese < ItaliansFrench < AmericansChinese < FrenchFrom the above, the only conclusion, that we can draw is "Chinese are shorter than Americans" (or) "Americans are taller than Chinese".                                                                                                                                                     Choice (D)

Page 13: JMET 2006 Original Paper

Directions for Questions 44 – 46: Answer Questions 44 – 46 based upon the following passage. Any additional information provided with a particular questions pertains to that individuals only. Eight entrepreneurs (J, K, L, M, N, O, P, Q) are selected for excellence awards for their outstanding contributions to business. Besides nurturing their own industries, two of these entrepreneurs, viz., J and L, also patronize the telecom industry while two others, viz., M and P, also patronize the computer industry. in arranging the seats, it was decided that the entrepreneurs who patronize other industries in addition to their own should not be seated together. 44. Which one of the following combinations is possible in the seating arrangements?

(A) KMLNJ                             (B) JPQOL                             (C) JKLNM                                  (D) JOLPQ Solution:Here, of the eight persons, J, L, M and P are the persons, who patronize other industries in addition to their own.So, no two of these persons are next to each other.And similar is the case with the remaining four persons i.e., K, N, O and Q. Hence, they must be in alternate positions.

 Choice (A) is not possible as L and M are in adjacent positions.Choice (B) is not possible as J and P are in adjacent positions.Choice (D) is not possible as L and P are adjacent to each other.Choice (C) is a valid arrangement.                                                                                                                     Choice (C)

 45. In order to ensure a proper seating arrangement, M should sit between

(A) N and O                           (B) K and J                            (C) L and N                                 (D) O and P

 Solution:Here, of the eight persons, J, L, M and P are the persons, who patronize other industries in addition to their own.So, no two of these persons are next to each other.And similar is the case with the remaining four persons i.e., K, N, O and Q. Hence, they must be in alternate positions.

 M should sit between N and

O.                                                                                                                         Choice (A) 46. Which one of the following can be seated next to O?

(A) J only                              (B) Q only                              (C) K only                                    (D) Any of the above Solution:Here, of the eight persons, J, L, M and P are the persons, who patronize other industries in addition to their own.So, no two of these persons are next to each other.And similar is the case with the remaining four persons i.e., K, N, O and Q. Hence, they must be in alternate positions. J can sit next to

O.                                                                                                                                         Choice (A)47. Harry is younger than Latif; Mahesh is of the same age as Latif; Randeep is exactly five yeas

older than Mahesh, and Sonam is younger than Randeep but older than Harry by at the most four years. What is the minimum difference between the ages of Randeep and Harry? Assume that ages of all persons in this question are integer values.

Page 14: JMET 2006 Original Paper

(A) four years                        (B) five years                         (C) six years                         (D) eight years

 Solution:Here, we are looking for the minimum possible difference between the ages of Randeep and Harry.As the difference between the ages of Mahesh and Randeep is five and the minimum possible difference between Mahesh, who is of the same age as Latif and Harry, who is younger than Latif is one.Total difference =

6.                                                                                                                                       Choice (C)Directions for Question 48: The following question has a set of five statements. Each statements has three sentences. Choose the alternative where the third sentence can be logically deduced, using both the preceding two, but not just from one of them. 48. (i)   Mild inflation is good for the economy. It encourages economic growth. Higher economic

growth rate leads to higher level of       inflation.(ii)  X and Y are friends. X is the enemy of Z. Y is the enemy of Z.(iii) Outsourcing of certain jobs improves the focus of the firm in the core area. It also raises

productivity and cost efficiency. Firms which outsource experience an increase in their productivity.

(iv)  Global trade is expanding very fast. It is for the benefit of developed countries to expand their trade faster with the developing countries rather than among themselves. Most of such trade is largely concentrated among developed countries.

(v)  Forgiveness is a great virtue. Great virtues are rare. Forgiveness is rare.(A) Only (i)                       (B) (i) and (ii)                         (C) (iii) and (v)                             (D) (iv) and (v)

 Solution:Here, we have to find the sets in which the third statement is the conclusion from the first two statements.(i)   The third statement cannot be a conclusion as the premises are talking about mild inflation but

conclusion is about high level of inflation.(ii)  The third statement cannot be a conclusion from the first two, as friend's enemy need not be the enemy of Y.(iii) This is a correct set, as the third statement is a correct conclusion from the preceding two.(iv)  This is not a correct conclusion as the conclusion is talking about the concentration of global

trade, whereas the premises are talking about benefits of global trade.(v)  It is a correct set as the conclusion is following the premises.Hence, (iii) and (v) are correct.                                                                                                                          Choice (C)

Directions for Questions 49 – 51 : Read the given passage and answer the questions: Six products, namely U, V, W, X, Y and Z are to be placed in the display windows of a shop. There are six display windows numbered 1, 2, 3, 4, 5 and 6 from left to right in such a way that one product is showcased in one window only. However, U cannot be placed adjacent to V. W must be immediately to the left of X and Z cannot be in window number 6. Note: The direction (left or right) should be determined with respect to the observer/shopper. 49. Which of the following products CANNOT   be placed in window number 1?

(A) U                                     (B)V                                            (C) W                                         (D) X 

Solutions: Here one should not be confused about the statement given in bold letters. As the answers are not going to change, whether we consider the directions based on the observer or not.The given conditions are

Page 15: JMET 2006 Original Paper

(i)   U cannot be adjacent to V.(ii)  W must be to the immediate left of X.(iii) Z cannot be in window number 6.As X has W to its left, it cannot be placed in window number

1.                                                                           Choice (D) 50. If X is placed in window number 3, in which window can W be placed?

(A)1                                      (B) 2                                           (C) 4                                          (D) 5 

Solutions: Here one should not be confused about the statement given in bold letters. As the answers are not going to change, whether we consider the directions based on the observer or not.The given conditions are(i)   U cannot be adjacent to V.(ii)  W must be to the immediate left of X.(iii) Z cannot be in window number 6.

 Given, X is placed in 3.As W is to the immediate left of X, W must be in window number 2.                                                                     Choice (B)

 51. If U is placed in window number 5, which of the following products must be placed in window number 6?

(A) V                                     (B) W                                          (C) Y                                           (D) X 

Solutions: Here one should not be confused about the statement given in bold letters. As the answers are not going to change, whether we consider the directions based on the observer or not.The given conditions are(i)   U cannot be adjacent to V.(ii)  W must be to the immediate left of X.(iii) Z cannot be in window number 6.

 Given, U is placed in window number 5.As V is not adjacent to U, V cannot be in window number 6.Given, Z is not in window number 6.As W and X are next to each other, none of them must be in window number 6.Y must be in window number 6.                                                                                                                     Choice (C)

52. All practicing lawyers have LLB degree. All MBBS degree holders are doctors. No doctors have LLB degree.

From the above statement, which one of the options can be logically deduced?(A) Every individual is either a practicing lawyer or does not have a MBBS degree(B) Every individual is either not a practicing lawyer or does not have a MBBS degree(C) Every individual is either not a practicing lawyer or has a MBBS degree(D) Every individual is either a practicing lawyer or has a MBBS degree

 Solution:Given,All practicing lawyers have LLB degree.No doctors have LLB degree.From these two, we can conclude thatNo practicing lawyer is a doctor.Further, the third premise given isAll MBBS degree holders are doctors.Combining the conclusion we got from the previous statement with the third statement.The final conclusion is

Page 16: JMET 2006 Original Paper

No practicing lawyer has MBBS degree.Of the given choices, only (B) represents this statement.                                                                                    Choice (B)

53. A family comprises seven members namely M, N, O, P, Q, R and S. Among them four are adults and three are children. Of the three children, only R and S are girls. M and P are brothers and M is a pilot. Q is an airhostess married to one of the brothers and has two children. N is married to P and S is their child. Who is O?(A) M’s son                      (B) R’s father                         (C) Q’s daughter                     (D) P’s son

 Solution:Given,M and P are brothers. As N is married to P, Q must have married to the remaining brother i.e., M.Of the three children, S is the child of P.Hence, R and O are children of M.As only R and S are girls, O must be a boy and hence he is the son of M.                                                           Choice (A)

54. If C $ D means C is the brother D, C * D means C is the mother of D, and C # D means C is the sister of D, which of the following would mean ‘M is the uncle of N’?(A) M $ C # N                        (B) N # M $ C                        (C) M $ C * N                         (D) M # C * N

 Solution:From the choices,(A) M $ C # NM is brother of C, who is sister of N. M is brother of N.(B) N # M $ CN is sister of M, who is brother of C. M is brother of N.(C) M $ C  NM is brother of C, who is mother of N. M is uncle of N.(D) M # C  NM is sister of C, who is mother of N. N is nephew / niece of M.                                                                                                                             Choice (C)

55. If the code for the word ‘QUESTION’ is SWGUVKQP, identify the option which gives the correct code for the word ‘RECEPTIONIST’?(A) TGEGRKVQPKUV           (B) TGEGRVQKPKUV           (C) TGEGRVKQPKUV           (D) TGERVKQKPUV

 Solution:Here in the coding, for every letter, we are writing its alternate letter as the code.For eg:-For Q, we are leaving R and we are considering the next letter S as its code.Similarly, RECEPTIONIST is coded as TGEGRVKQPKUV.                                                                                Choice (C)

56. Ram, Qadir, Sorabji, Charles and Gurdeep all roll the same unbiased dice one after the other and note down their respective numbers. Ram gets an even number; Qadir gets a number greater than that of Gurdeep; Sorabji gets the same number as Charles and the addition of Charles’ and Ram’s numbers is odd. If Qadir’s number is 4, what is the maximum of the addition of the numbers obtained by all five?(A) 22                                    (B) 23                                    (C) 25                                         (D) 27

 Solution:It is given that sum of Charle’s and Ram’s number is odd and Ram gets an even number. Hence, the Charle’s number must be odd. A dice will have any one of the number among 1, 2, 3, 4, 5 or 6.The maximum sum is to be calculated. Hence, the maximum possible numbers are to be considered.Ram      even number      6

Page 17: JMET 2006 Original Paper

Qadir     (given)               4Charles        odd             5Sorabji   same as Charles     5Gurdeep      less than Qadir  3Hence the total can be6 + 4 + 5 + 5 + 3 = 23.                                                                                                                                    Choice (B)

Answer Questions 57 – 60 based upon the following passage. U, V, X, Y and Z collected CDs of yesteryears’ melodious songs. They collected a total of 100 CDs. None of them have collected less than 10 CDs. No two among them collected the same number of CDs. Also,(i)   U collected the same number of CDs as V and X put together.(ii)  X collected 3 more CDs than the cube of an integer.(iii) The number of CDs collected by U was the square of an integer.(iv)  The number of CDs collected by V was either the square or the cube of an integer.(v)  The number of CDs collected by Y and Z are in ratio 4:3. 57. What was the number of CDs collected by U?      (A) 19                                    (B) 36                                     (C) 52                                    (D) 64 

Solution:Given,U = V + XX = Cube +3U = SquareV = Square / CubeY : Z = 4 : 3X = Cube +3The only possibilities are 11 or 30, the other (cube +3) values are not possible as each of the five persons has collected at least 10 CDs.If X = 30, for any value of U, we will not get V as a square or cube.Hence, X = 11; U = 36; V = 25; Y = 16; Z = 12                                                                                                  Choice (B)

 58. What was the number of CDs collected by V?      (A) 16                                     (B) 25                                    (C) 46                                    (D) 64 

Solution:Given,U = V + XX = Cube +3U = SquareV = Square / CubeY : Z = 4 : 3X = Cube +3The only possibilities are 11 or 30, the other (cube +3) values are not possible as each of the five persons has collected at least 10 CDs.If X = 30, for any value of U, we will not get V as a square or cube.Hence, X = 11; U = 36; V = 25; Y = 16; Z = 12                                                                                                  Choice (B)

 59. What was the difference in the number of CDs collected by X and Y?      (A) 5                                     (B) 7                                     (C) 9                                     (D) 11 

Solution:Given,U = V + XX = Cube +3U = Square

Page 18: JMET 2006 Original Paper

V = Square / CubeY : Z = 4 : 3X = Cube +3The only possibilities are 11 or 30, the other (cube +3) values are not possible as each of the five persons has collected at least 10 CDs.If X = 30, for any value of U, we will not get V as a square or cube.Hence, X = 11; U = 36; V = 25; Y = 16; Z = 12                                                                                                  Choice (A)

 60. How many of the individual collects(s) of CDs was / were prime numbers?      (A) 0                                (B) 1                                       (C) 2                                      (D) 3 

Solution:Given,U = V + XX = Cube +3U = SquareV = Square / CubeY : Z = 4 : 3X = Cube +3The only possibilities are 11 or 30, the other (cube +3) values are not possible as each of the five persons has collected at least 10 CDs.If X = 30, for any value of U, we will not get V as a square or cube.Hence, X = 11; U = 36; V = 25; Y = 16; Z = 12                                                                                                  Choice (B)

61. Not only do major diseases afflict the elderly in particular – of which there are more – they are generally very expensive diseases to treat. Which one of the following options, if true, MOST STRENGTHENS this statements?(A) Studies reveal that 60% of elderly people suffer from cataract(B) A dentist’s major income comes from fixing artificial teeth for elderly patients(C) Insurance companies do not cover Parkinson’s disease because of the expenses involved(D) Private nursing homes have reported that cancer accounts for 50% of prolonged hospitalization of the elderly

 Solution:We have to look for the statement that most strengthens the given statement.Statement (A) is not the required one as it talks about only suffering but not the expenses associated with it.Statement (B) will not strengthen the given statement as fixing artificial teeth is not a disease at all.Statement (D) will not strengthen the given statement as it is talking about period rather than the expenses involved with hospitalisation.Statement (C) will strengthen the given argument as it is talking about the expenses involved with the disease.       Choice (C)

Directions for Questions 62 – 65: Answer Questions 62 – 65 based upon the following passage. Any additional information provided with a particular question pertains to that individual question only. The placement office of a management school has to schedule seven companies denoted by P to V for the placement week (the week starts on a Sunday) but not necessarily in that order. This group of seven companies is divided into three categories, namely, finance, manufacturing and sales. The number of companies in these three categories are three, two and two respectively. The placement schedule is such that the same category companies have to be scheduled consecutively. It has also been decided that:(i)   Company S, a manufacturing company, cannot be scheduled on the sixth day of the week.(ii)  Companies Q and U are scheduled consecutively but not necessarily in that order.(iii) Companies T can be scheduled on any day of the week except on Saturday.(iv)  Company R, which is a similar category company as Q and U, has to be scheduled on the third day of the week.

Page 19: JMET 2006 Original Paper

 62. The total number of schedules, the placement office can have, is

(A) 36                                    (B) 48                                          (C) 56                                         (D) 64 

Solution: The names of the seven companies are P, Q, R, S, T and U.From (iv), R, Q and U belongs to the same category, finance.From (ii) and the rule that the same category companies have to be scheduled consecutively, we can have Q, R and U in three consecutive days, in any order and further, Q and U must be on consecutive days.

 From (i), (iii) and (iv) and the above results, the possibilities are(a)  1    –    Q/U           Q/U           Q/U      2    –    U/Q           U/Q           U/Q      3    –    R         R         R      4    –    S         __        __      5    –    __        S         __      6    –    __        __        __      7    –    __        __        S                 (i)        (ii)        (iii) Again from (i)

       Q/U           Q/U      U/Q           U/Q      R         R      S         S      T         P/V      P/V      T      V/P      V/P                       (4)       (4)            Possibilities 

From (ii) Q/U           Q/U

      U/Q           U/Q      R         R      S         S      T         P/V      P/V      T      V/P      V/P                       (4)       (4)                 Possibilities 

From (iii)       Q/U           Q/U           Q/U      U/Q           U/Q           U/Q      R         R         R      V/P      P/V      T

P/V      T         P/V      T         V/P      V/P      S         S         S                              (4)       (4)       (4)       Possibilities (b)  S         __        __

Page 20: JMET 2006 Original Paper

      __        S         __      R         R         R      Q/U           Q/U           Q/U      U/Q           U/Q           U/Q      __        __        __      __        __        S       (i)        (ii)        (iii)       Again from (i)       S         S      T         P/V      R         R      Q/U           Q/U      U/Q           U/Q      P/V      T      V/P      V/P       (4)       (4)             Possibilities       Again from (iii)       T         V/P      S         S      R         R      Q/U           Q/U      U/Q           U/Q      P/V      T      V/P      P/V       (4)       (4)             Possibilities 

Again from (iii)       P/V           P/V           T      V/P           T                P/V      R               R               R      Q/U           Q/U           Q/U      U/Q           U/Q           U/Q      T         V/P           V/P      S         S         S       (4)       (4)       (4) Hence, a total of 4  14 = 56 schedule.                                                                                                                   Choice (C) 63. If P is scheduled on Wednesday, which of the following has to TRUE?

(A) U has to be scheduled on Thursday(B) V has to be scheduled either on Friday or Saturday(C) T has to be scheduled either on Thursday or Friday(D) S is always scheduled on some day of the week before Q is scheduled

 Solution: The names of the seven companies are P, Q, R, S, T and U.From (iv), R, Q and U belongs to the same category, finance.

Page 21: JMET 2006 Original Paper

From (ii) and the rule that the same category companies have to be scheduled consecutively, we can have Q, R and U in three consecutive days, in any order and further, Q and U must be on consecutive days. If P is scheduled on Wednesday i.e., on the fourth day, then the possible arrangement can be

                  Now, from the choices,Choice (A) cannot be true as U can be scheduled on Sunday or Monday. 

Choice (B) cannot be true as                     is a possible arrangement. Choice (C) can be true as T cannot be placed on Saturday, the left over places are Thursday and Friday.Choice (D) cannot be true as S cannot be placed before Q.                                                                                Choice (C)

 64. If S is scheduled on Monday, which one of the following options is definitely FALSE?

(A) P and T are sales companies.(B) The maximum number of days in between the schedules of T and U is three.(C) R is always scheduled before Q.(D) The number of schedules when P is slotted on Sunday is three.

 Solution:The names of the seven companies are P, Q, R, S, T and U.From (iv), R, Q and U belongs to the same category, finance.From (ii) and the rule that the same category companies have to be scheduled consecutively, we can have Q, R and U in three consecutive days, in any order and further, Q and U must be on consecutive days.

 

If S is scheduled on Monday, then the arrangement can be Now, from the choices,

Choice (A) can be true, as    is a possible arrangement and it is given that S is a manufacturing company and hence P and T can besales companies.

Choice (B) can be true, as the arrangement    is possible and hence the maximum number of days between the schedules of T and U is three.Choice (C) is always true.Choice (D) can never be true as P  S  R  Q  U  T  V(or) P  S  R  U  Q  T  V are the only the possibilities.                                                                                          Choice (D)

 65. If T is a sales company and S is scheduled on Saturday, which of the following schedules are possible?

(i)   U-Q-R-T-P-V-S (ii)  P-T-R-U-Q-V-S(iii) T-V-R-Q-U-P-S (iv)  Q-U-R-P-V-T-S(A) (i)-(iii)-(iv)                          (B) (ii)-(iii)-(iv)                         (C) (i)-(ii)-(iv)                          (D) (i)-(ii)-(iii)

 Solution:The names of the seven companies are P, Q, R, S, T and U.From (iv), R, Q and U belongs to the same category, finance.

Page 22: JMET 2006 Original Paper

From (ii) and the rule that the same category companies have to be scheduled consecutively, we can have Q, R and U in three consecutive days, in any order and further, Q and U must be on consecutive days.If S is scheduled on Saturday and it is a manufacturing company and T is a sales company implies T cannot be placed before S, as the same category companies have to be scheduled consecutively. So, (iv) cannot be true. By eliminating the options, it is evident that choice (D) is the only possible one.                                                                                              Choice (D)

Directions for Questions 66 – 67: The statements in questions 66 – 67, when properly sequenced, form a coherent paragraph. Each sentence is labelled with a number. Select the most logical order of the sentence in each case. 66. (i)   This is the large part due to the dominance of a view within the leadership literature, popular

management theory and the       media that there is something special about leadership;(ii)  Leadership as a phenomenon, we are often told, transcends the everyday, the mundane and the ordinary;(iii) Yet few studies have ventured into the everyday doing of leadership-particularly within an educational setting;(iv)  The need to conduct more detailed studies of leadership-in-practice has long been

recognized in both leadership studies and educational research;(A) (iv)-(i)-(ii)-(iii)                      (B) (iv)-(iii)-(i)-(ii)                      (C) (ii)-(iii)-(iv)-(i)                      (D) (ii)-(iv)-(iii)-(i)

 Solution:Ideally statement (ii) should begin the paragraph because it is a generalized statement that can begin a paragraph. Statement (iii) which begins with ‘yet’ counters the assumption stated in (ii) and is therefore linked to (ii). Statement (iv) which states that more detailed studies of leadership should be conducted follows statement (iii) as it concludes the idea. Statement (i) which mentions why such a detailed study is essential concludes the paragraph. Hence (ii) – (iii) – (iv) – (i) is the correct order of sentences.                                                                                                                                                    Choice (C)

 67. (i)   Studies of successful marriages indicate that periods of fighting and turmoil require a lot of love and passion as            compensation for subversive episodes;

(ii)  Closely related is solid evidence that martial satisfaction is positively related to reliance on constructive problem solving strategies, mainly negotiation and compromise;

(iii) A solid and substantial sense of connection with other people requires a healthy balance between positive and negative feelings;

(iv)  In effect, more positive than negative energy is required to sustain intimate relations over time;(A) (iii)-(i)-(ii)-(iv)                      (B) (iii)-(ii)-(iv)-(i)                      (C) (ii)-(i)-(iv)-(iii)                      (D) (ii)-(iii)-(iv)-(i)

 Solution:Statement (iii) is the introductory statement as it mentions the idea on which the paragraph is based. Statement (i) corroborates what is stated in (iii) by citing an illustration of successful marriages. Statement (ii), which states marital satisfaction is positively related to reliance on problem solving strategies like negotiation and compromise, is a continuation of (i). Statement (iv) which declares that more positive energy rather than negative energy to sustain intimate relations, ideally concludes the para.                                                                                                                                                              Choice (A)

Directions for Questions 68 – 69: Read the following information carefully and answer the questions. 

Page 23: JMET 2006 Original Paper

(i)   In a newly constructed municipality market in Nagpur, six shops on a floor in two rows facing North and South are allotted to U, V, W, X, Y and Z, not necessarily in that order. The number of shops on both the rows are equal in number.

(ii)  V gets a North-facing shop and is not next to X.(iii) X and Z get diagonally opposite shops.(iv)  W which is next to Z gets a South-facing shop and Y gets a North-facing shop. 68. Whose shop is between V and X?

(A) Z                                      (B) Y                                     (C) W                                    (D) U 

Solution:From (i), the number of shops on both the rows are equal in number implies that three shops are to be placed in each row. From (ii), V gets North-facing shop. From (iv), W gets a South-facing shop and Z is next to it implies Z also gets a South-facing shop. From (iii), The shops of X and Z are not in the same row and from the above point, we can say that X gets a North-facing shop. From (iv), Y gets a North-facing shop hence U gets a South-facing shop. So,South-facing shops – (Z, U, W)North-facing shops – (V, X, Y). Z    W  U         U   W  Z      (or)V   Y   X         X    Y   V

 Y’s shop is in between V and

X.                                                                                                                        Choice (B) 69. The shops of which of the other pairs, apart from X and Z are diagonally opposite to each other?

(A) U and Y                           (B) V and U                           (C) V and W                          (D) Y and X

 Solution:From (i), the number of shops on both the rows are equal in number implies that three shops are to be placed in each row. From (ii), V gets North-facing shop. From (iv), W gets a South-facing shop and Z is next to it implies Z also gets a South-facing shop. From (iii), The shops of X and Z are not in the same row and from the above point, we can say that X gets a North-facing shop. From (iv), Y gets a North-facing shop hence U gets a South-facing shop. So,South-facing shops – (Z, U, W)North-facing shops – (V, X, Y). Z    W  U         U   W  Z      (or)V   Y   X         X    Y   V

 Apart from X and Z, the shops of U and V are diagonally opposite to each

other.                                                   Choice (B)70. A group comprising five persons namely M, N, O, P and Q includes one professor of civil

engineering and one of mechanical engineering. In the group, M and P are unmarried women. No woman is either a chemical or a mechanical or a civil engineer. There is just one married couple in the group with Q as the husband. N is the brother of O and is neither a chemical engineer nor a mechanical engineer. What is the profession of Q?

      (A) Civil Engineer               (B) Chemical Engineer            (C) Mechanical Engineer         (D) Cannot be determined 

Solution:Given that M and P are unmarried women and no women is either a chemical or a mechanical or a civil engineer. Hence Neither M nor P is either a civil engineer or mechanical engineer.

      It is given that Q is the husband of one among the group. Hence Q is male and also N is the brother of O. Hence, N is also a male and M and P are unmarried. Hence, O can be the wife of Q. As O is a female, she cannot be either chemical or a mechanical or a civil engineer. Hence either N or Q can be the civil engineer or the mechanical engineer, in any order. At the end it is given

Page 24: JMET 2006 Original Paper

that N is not a mechanical engineer. Hence Q must be the mechanical engineer.                             Choice (C)

Directions for Questions 71 – 72:

 

Each of the problems in this section contains a question and two statements which are labelled as (1) and (2). Use the information provided in statement (1) and (2) and the corresponding question to decide whether the statements are sufficient to answer the question. For each problem, determine which of the following is the CORRECT alternative:

 

71. Can we find out the value of the eleventh number in a set of eleven numbers, if

(1)  the average of the first ten number in the set is given

(2)  the average of all the eleven numbers is given

      (A) (1) and (2) together are not sufficient

(B) (1) and (2) taken together are sufficient, but neither (1) alone, nor (2) alone is sufficient

(C) (1) alone is sufficient, but (2) alone is not sufficient

(D) (2) alone is sufficient, but (1) alone is not sufficient

 

Solution:

Using  alone,

      If the average of the first ten numbers in the set is given, then we can be able to find the sum of the first ten numbers only. Hence it is not sufficient.

      Using  alone,

      If the average of eleven numbers is given, we can be able to find the sum of the eleven numbers only. Hence it is also not sufficient.

      Using  and  together.

      (Sum of the eleven numbers)  (sum of the first ten numbers) = the eleventh number.                                            Choice (B)

 

72. Can we find the relative speed of two trains with respect to each other, if

(1)  the speed of the first train is 120% more than the speed of the second train

      (2)  the speed of the second train is 80 km/hr

      (A) (1) and (2) together are not sufficient

(B) (1) and (2) taken together are sufficient, but neither (1) alone, nor (2) alone is sufficient

(C) (1) alone is sufficient, but (2) alone is not sufficient

Page 25: JMET 2006 Original Paper

(D) (2)  alone is sufficient, but (1) alone is not sufficient

 

Solution:

Using  alone,

      The speed of the first train is 120% more than the speed of the second train. It implies the ratio of speeds of the first train to the second train will be 11:5. But the relative speed of two trains cannot be determined. Hence, it is not sufficient.

      Using  alone,

      The speed of the second train is 80 km/hr. Clearly the statement alone is not sufficient.

      Combining  and ,

      We can say the speeds of the trains. But in order to calculate the relative speed of the trains, the directions in which they are travelling must be known and it is not given.

      Hence  and  together are also not sufficient.                                                                                                    Choice (A)

73. Starting from his office, Fernandez drives his car towards the North for 40 km. He than takes a right and travels for 30 km to reach ‘VXL Petrol Pump’. From there, he again drives North-West for a distance of 50 km, before travelling North for another 40 km. Finally he turns and travels towards South-West for 50 km and stops. What is his direction now with respect to the starting point?

      (A) South-West                      (B) North-East                       (C) North-West                      (D) South-East

 

Solution:

The path traversed by Fernandez is as follows.

 

 

 

 

Page 26: JMET 2006 Original Paper

 

 

 

 

 

 

 

 

      Hence, he is towards North-west with respect to the starting point.                                                                      Choice (C)

Directions for Questions 74 – 75:

 

Each question contains six statements followed by four sets of combinations of three. Choose the set in which the statements are logically related.

 

74. (i)   Reena and Meena and sisters

      (ii)  Sisters are often known to quarrel

      (iii) Reena and Meena do not quarrel

      (iv)  All those who quarrel are sisters

      (v)  Reena and Meena quarrel often

      (vi)  Reena and Meena cannot be sisters

      (A) (i)-(iv)-(v)                      (B) (i)-(iv)-(vi)                     (C) (iii)-(iv)-(v)                    (D) (i)-(ii)-(v)

 

Solution:

Choice (A) is not a valid option, as there are four terms.

      Choice (B) is not a valid option as there is only one negative statement.

      Choice (C) is not a valid option as there is only one negative statement.

      Choice (D) is a valid option as it did not violate any of the conditions.                                                                   Choice (D)

 

75. (i)   All crows are birds

      (ii)  All birds are not crows

Page 27: JMET 2006 Original Paper

      (iii) All birds are warm blooded

      (iv)  All crows lay eggs

      (v)  All birds lay eggs

      (vi)  Crows are warm blooded

      (A) (i)-(iv)-(v)                      (B) (ii)-(v)-(iv)                     (C) (i)-(ii)-(v)                      (D) (i)-(iii)-(vi)

 

Solution:

Choice (A) is a valid option when taken in the order (i) – (v) – (iv).

Choice (B) is not a valid option as there is only one negative statement.

Choice (C) is not a valid option as there is only one negative statement.

Choice (D) is a valid option when it is considered in the same manner it self.

      (A) and (D) are both logically related but among these two, D is the most appropriate choice as the order is also correct in D. 

                                                                                                                                                                            Choice (D)

Directions for Questions 76 – 78: Read the following information and answer the questions.

Any additional information provided with a particular question pertains to that individual

question only.

 

The production, marketing, human resource, finance, and management information system managers of a particular company meet for a round table meeting to discuss the strategy of the company. Out of the five, three are men and the other two are women. The following restrictions apply to their seating arrangement:(i)   The two women will not be seated next to each other.

(ii)  The production manager, a man, will always be seated as far as possible from the marketing manager.

(iii) The finance manager will always be seated next to a woman.

 

76. If the human resource manager is a man, which of the following is definitely NOT true?

(A) The marketing manager is sitting in between two women  (B) The marketing manager is sitting in between two men

      (C) The finance manager is a man                                       (D) the marketing manager is a woman

 

      Solution:

Page 28: JMET 2006 Original Paper

It is given that, Production Manager will always be seated as far as possible from the marketing manager. Let us represent this by representing men by (+) and women by (–).

            If HR Manager is a man. The three men are managers of HR, Finance and Production hence the marketing manager and the manager of Management Information Systems (MIS) are women.

The possibilities are 

       

Hence, of the given statements, only A is not true.                                                                                             Choice (A)

 77. If the finance manager always has a woman to his right, in how many different arrangements can

the marketing manager be a woman?

      (A) 2                                     (B) 3                                     (C) 4                                     (D) 5

     

      Solution:

It is given that, Production Manager will always be seated as far as possible from the marketing manager. Let us represent this by representing men by (+) and women by (–).

Page 29: JMET 2006 Original Paper

      

      The Finance Manager has a woman to his right, that woman can be a Marketing Manager or HR / Finance Manager. The possibilities are as follows.

      (i)       

      (ii)        

(iii)                 A total of three

possibilities.                                                                                                                       Choice (B) 

78. If the management information system manager is sitting just to the right of the production manager, which of the following statement(s) is/are definitely TRUE?

Page 30: JMET 2006 Original Paper

(i)   The marketing manager and the finance manager sit next to each other five times.

(ii)  The human resource manager has to be a woman.

(iii) The management information system manager has to be a man.

      (A) (i) only                             (B) (i) and (ii)                         (C) (i) and (iii)                        (D) (ii) and (iii)

 

      Solution:

It is given that, Production Manager will always be seated as far as possible from the marketing manager. Let us represent this by representing men by (+) and women by (–).

      

      The possibilities are

       

     

       From the given choices,

Page 31: JMET 2006 Original Paper

      (ii) and (iii) are not true.      As (i) is a possibility, (i) is

true.                                                                                                                        Choice (A)79. In a meeting of board of directors of a company ridden with financial crisis, one of the directors

said, “I firmly believe that the only way to tide over the current financial crisis, due to heavy erosion in profit, is to bring out a new series of product versions. Such actions on the part of the company will meet the requirements of various market segments, thus raising the sales volume and profit. I challenge anyone who disagrees with my opinion to prove why this suggestion should not help the company to overcome the crisis.”

 

The MOST EVIDENT logical flaw in the director’s statement is that

(A) It fails to provide any statistical evidence in support of his contention

(B) It shifts the onus of proof to those could object to his opinion

(C) It does not talk about employee dissatisfaction that might have put the company in the current financial mess

(D) It works on the assumption that the only reason for erosion in profits is lack of large number of product versions

 

Solution:The most logical flaw in the given argument is the person is asking his contenders to prove the argument, rather than proving its correctness by himself.                                                                                                                                    Choice (B)

80. A study reveal that software firms that do not make investments in training their employees report high attribution rates and low employee productivity. Which one of the following options, if true, would MOST WEAKEN this statements?

(A) Studies regarding employee turnover rates in manufacturing organizations have reported similar findings

(B) Employee-friendly policies in organizations help to improve commitment levels of employees

(C) A high correlation has been found between high salary levels and high employee productivity

(D) Organizations regularly lose their highly trained and productive employees to their competitors

 Solution:Of the given choices, (D) is the one which weakens the given statements the most that too in two ways. One is it is a generalized statement stating that all the organizations will lose employees regularly. The second one is organizations lose the trained employees, hence the firms mentioned in the question cannot retain the employees by giving them training.                                                                                                                                                                     Choice (D)

SECTION 3: QUANTITATIVE ABILITY (Questions 81 – 120) 81. If the price of sugar is raised by 25%, by how much should a household reduce its consumption of

sugar so as not to increase expenditure on sugar?      (A) 20%                                  (B) 25%                                 (C) 30%                                  (D) 35% 

Page 32: JMET 2006 Original Paper

Solution:Initially, let the household consumption of sugar be 10 kg and let the cost of the sugar be Rs.10 per kg.

      The expenditure on sugar = Rs.100      Now, the price of the sugar has increased by 25% i.e. new price is Rs.12.5 per kg.

      For Rs.100, the household can now buy   i.e. 8 kg of sugar.

      So, the consumption should come down by   i.e. 20% to maintain the same expenditure.                  Choice (A)

82. The age of Mr. Chetan in 2002 was   of his birth year. What is his age in 2006?      (A) 30                               (B) 28                                     (C) 26                                    (D) 22 

Solution:From the question, it is clear that the year of birth of Mr. Chetan is a multiple of 90. The nearest multiple of 90 less than 2002 is 1980.

      If we take any lesser multiple of 90, we don’t get any feasible solution.      So, the age of Mr. Chetan in the year 2006 is 2006–1980 i.e. 26

years.                                                                 Choice (C)83. x2 – 2x + y2 – 4y + 5 = 0 on the xy – plane represents      (A) a point                        (B) a circle                        (C) an ellipse                    (D) a hyperbola 

Solution;Given, x2 – 2x + y2 – 4y + 5 = 0

       (x – 1)2 + (y – 2)2 = 0       x – 1 = 0 and y – 2 = 0       x = 1 and y = 2

 The equation x2 – 2x + y2 – 4y + 5 = 0 represents the point (1, 2) in the xy – plane.                                          Choice (A)

Questions 84 and 85 are based on the following:

 

The cost of fuel for running the engine of an army tank is proportional to the square of the speed and Rs.64 per hour for a speed of 16 kmph. Other costs amount to Rs.400 per hour. The tank has to make a journey of 400 km at a constant speed. 

84. The most economical speed for this journey is

      (A) 20 kmph                          (B) 30 kmph                          (C) 35 kmph                          (D) 40 kmph

 

      Solution:

Let s be the speed of the tank.

The time taken by the tank to cover a distance of 400 km distance = 

No, C  s2 (Given)

 C = ks2, here k = constant.

Page 33: JMET 2006 Original Paper

But, C = 64, when s = 16

 64 = k (16)2  k = 

Cost of fuel for one hour = 

 The total cost of the fuel is Rs.

Other cost per hour = Rs.400

Total other costs = Rs.

So, the total cost (C) = 

Now for minimum cost, 

    s = 40

 

      The most economical speed is 40 kmph.                                                                                                          Choice (D)

 

85. The total cost for the journey at this most economical speed is

      (A) Rs. 6000                          (B) Rs.8000                           (C) Rs.10000                         (D) Rs.11000

 

      Solution:

Let s be the speed of the tank.

The time taken by the tank to cover a distance of 400 km distance = 

No, C  s2 (Given)

 C = ks2, here k = constant.

Page 34: JMET 2006 Original Paper

But, C = 64, when s = 16

 64 = k (16)2  k = 

Cost of fuel for one hour = 

 The total cost of the fuel is Rs.

Other cost per hour = Rs.400

Total other costs = Rs.

So, the total cost (C) = 

Now for minimum cost, 

    s = 40

 

The total cost at the economical speed =   + 100(4) = 4000 + 4000 = Rs.8000                                         Choice (B)

86. A textile manufacturing firm employs 50 looms. It makes fabrics for a branded company. The aggregate sales value of the output of the 50 looms is Rs.5,00,000 and the monthly manufacturing expenses is Rs.1,50,000. Assume that each loom contributes equally to the sales and the manufacturing expenses are evenly spread over the number of looms. Monthly establishment charges are Rs.75,000. If one loom breaks down and remains idle for one month, the decrease in profit is

      (A) Rs.13000                         (B) Rs.10000                         (C) Rs.7000                           (D) Rs.5500

 Solution:Sales value of the output per loom = Rs.10,000 per month

Monthly manufacturing expense per loom =  = Rs.3000The establishment expense doesn’t depend on the number of looms in operation as it remains the same irrespective of the number of looms in operation.As one of the looms was kept idle, the revenue is reduced by Rs.10,000 and the manufacturing expense reduced by Rs.3000. The overall profit reduced by

Page 35: JMET 2006 Original Paper

(10000 – 3000) i.e. Rs.7000.                                                                                                                             Choice (C)

87. Two straight lines can divide a circular disk into maximum of 4 parts. Likewise, into how many maximum parts can four straight lines divide a circular disk?

      (A) 8                                     (B) 9                                     (C) 10                                   (D) 11

 

Solution:

4 lines divided a circular disc into a maximum of 11 parts as shown below.

 

 

 

 

 

 

 

 

 

 

Note: 4 lines divide a plane into a maximum of 11 regions. We can certainly draw a circle which encloses a part of all these regions.                                                                                                                                                          Choice (D)

88. The currencies in countries X and Y are denoted by Xs. and Ys. respectively. The exchange rate in 1990 was 1 Xs. = 0.6 Ys. The price level in 2006 in X and Y are 150 and 400 respectively with 1990 as a base of 100. The exchange rate in 2006, based solely on this purchasing power parity consideration, is 1 Xs. =

      (A) 0.225 Ys.                         (B) 0.625 Ys.                              (C) 1.6 Ys.                            (D) 3.6 Ys.

 Solution:It is given that, the prices in country X increase from 100 to 150.

      It also given that, the value of any currency depends on its purchasing power.

Page 36: JMET 2006 Original Paper

      As the prices are increasing, the purchasing power of the currency decreases. It decreases as follow.

                                            Similarly, the prices in country Y increased from 100 to 400.       the purchasing power (i.e. value) of currency of Y decrease as follows:

                                            In 1990,

        X = 0.6Y

      In 2006, the ratio is   =  = 8 : 5

       The required ratio is        1 Xs = 1.6

Ys                                                                                                                                             Choice (C)

89. In a family of husband, wife and a daughter, the sum of the husband's age, twice the wife's age, and thrice the daughters age is 85; while the sum of twice the husband's age, four times the wife's age, and six times the daughter's age is 170. It is also given that the sum of five times the husband's age, ten times the wife's age and fifteen times the daughter's age equals 450. The number of possible solutions, in terms of the ages of the husband, wife and the daughter, to this problem is

      (A) 0                                (B) 1                                      (C) 2                                      (D) infinitely many

 Solution:Let x, y and z (in years) be the ages of husband, wife and the daughter respectively.Then,x + 2y + 3z = 852x + 4y + 6z = 170and 5x + 10 y + 15z = 450     ----- (3)From (1) x + 2y + 3z = 855  (1) is 5x + 2y + 3z = 425But from (3) 5x + 2y + 3z = 450These two equations are inconsistent.Hence, the given problem has no solution.                                                                                                          Choice (A)

90. The determinant   equals to  (A) 1 + x1 + x2 + x3 + x4               (B) x1 + x2 + x3 + x4                (C) x1 x2 x3 x4   .                     (D) 1 +x1 x2 x3 x4

 Solution: 

      (Applying R1  R1 + R2 + R3 + R4), we get

Page 37: JMET 2006 Original Paper

= (1 + x1 + x2 + x3 + x4)   (Applying C2  C2 – C1, C3  C3 – C1, C4  C4 – C1), we get

(1 + x1 + x2 + x3) . 1 .    = (1 + x1 + x2 + x3) . 1 = 1 + x1 + x2 + x3 +

x4                                                                                               Choice (A)91. Ms.Rao paid equated monthly installments (EMIs) of Rs.25,000 each in January and February towards

her home loan, whose outstanding principal amount was Rs.10,00,000 in December. Each EMI consists of interest on outstanding loan amount for the month and part payment of the loan amount. If the interest on the loan is 12% per annum (interest is paid monthly on the diminishing outstanding loan), the total amount of interest that was paid by Ms.Rao in January and February was

      (A) Rs.30,150                        (B) Rs.20,000                        (C) Rs.19,850                        (D) Rs.19,700

 Solution:Rate of interest is 12% p.a. or 1% per month.

      Let the principal amount outstanding on December 1st be Rs.10,00,000.      Loan outstanding as on January 1st is Rs.10,00,000 (including) an interest of Rs.10,000).

Rs.25,000 is paid as the instalment and the loan outstanding becomes Rs.9,85,000. The interest added on February 1st is 1% of Rs.9,85,000 i.e. Rs.9,850.

       The total interest paid= Rs.10,000 + Rs.9,850

              (in Jan)   (in Feb)      =

Rs.19,850                                                                                                                                                    Choice (C)

92. log10 (log23) + log10 (log34)+    + log10 (log10231024) equals      (A) 10                                   (B) e                                     (C) 1                                     (D) 0 

Solution:log10(log23) + log10(log34) + ----- + log10(log1023 1024)= log10[log23  . log34 . log45 ----- .log10231024]

      = log10

           = log10    = log10(log21024)       = log10(log2 210) = log1010 = 1                                                                                                                            Choice (C)93. The angles of a convex hexagon in degrees are integers and in arithmetic progression.

Let M denote the largest of these 6 angles. Then the maximum value that M can take is

Page 38: JMET 2006 Original Paper

      (A) 125°                                   (B) 150°                                   (C) 175°                                 (D) 179°

 Solution:Let the six angles of a convex hexagon be (a – 5d), (a – 3d), (a – d), (a + d), (a + 3d) and (a + 5d) where d > 0.

      The sum of the angles of the hexagon = 6a = 720       a = 120      So, the greatest angle is 120° + 5d.      Since the angles when expressed in degrees are integers.      d is an integer.      As the hexagon is convex, all its angles are less than 180°.       120° + 5d < 180°       The greatest possible angle could be 175° (at d =

11).                                                                                    Choice (C)94. The smallest positive value of x for which the fractions

 are in their simplest form is (A) 47                                   (B) 49                                   (C) 51                                   (D) 53

 Solution:

The given series of fractions are 

 . . . , 

 The series can be written as 

 . . . . .

       For the given fractions to be in simplest form (x + 2) must be co-prime to 10, 11, 12, 13 . . . .   49

and 50.      So, the smallest value of (x + 2) is the least prime number greater than 50.       x + 2 = 53  x =

51                                                                                                                                   Choice (C)95. Two barrels contain a mixture of ethanol and gasoline. The content of ethanol is 60% in the first barrel

and 30% in the second barrel. In what ratio must the mixtures from the first and the second barrel be taken to form a mixture containing 50% ethanol?

      (A) 1 : 2                                  (B) 2 : 1                                  (C) 2 : 3                                 (D) 3 : 2 

Solution:

     

Page 39: JMET 2006 Original Paper

              

 60% ethanol and 30% ethanol must be mixed in the ratio of (50 – 30) : (60 : 50) i.e. 2 : 1 to get 50% ethanol.     Choice (B)

96. Area (in sq units) bounded by the line y = x and the parabola y = x (x – 2) is

      (A)                                    (B)                                       (C)                                   (D)  

Solution:We have, y = x(x–2) = x2 – 2x

       y = (x – 1)2 – 1 The region bounded by the curves y = x and y = x(x–2) is shaded in the figure below :

            

 Required area = 

= 3 .  – 9 =   sq. units.                                                                                                                            Choice (B)

97. A filter paper of the form of a right circular cone of base radius 20 cm and altitude 40 cm is placed with its axis vertical and the vertex downwards. Water flows out at the rate of 22.5 cc. The rate at which the level of the water falls when the depth of the water is 30 cm is

Page 40: JMET 2006 Original Paper

      (A)                                  (B)                                (C)                                    (D)  

Solution:

           Given      h = 40 cm.      r = 20 cm      clearly, height is double of the radius. h = 2r      Given,

        = 22.5      We know that,

      V = 

      V = 

      V = 

      

      22.5 =     [ ∵ Given, h = 30 cm]

      

 The rate of which water level falls is                                                                                                        Choice (A)98. An antenna stands in the middle of a square tower. A man on the ground, opposite the middle of the face

of the tower and at a distance of 100 m from its foot, just sees the top of the antenna; on receding

another 100 m, the tangents of elevation of the top of the tower and the antenna are found to be   

and   respectively. The ground being horizontal, the height of the antenna (in 

      meters) is (neglect the height of the person for computations)

      (A)                                     (B) 25                                   (C) 50                                   (D)  

Page 41: JMET 2006 Original Paper

Solution:             

The man can just see the top of the antenna from P, 100 m from the foot of the tower F. M is the point directly below the antenna. i.e., the centre of the base and N in the centre of the top of the tower.Given tanBQF (not shown in the figure)= 1/2

       BF = 100 and BPF = ABN 45°       Height of antenna AN = BN = FM = x(say)      Given tan AQM = 5/9

               x = 25                                                                                                                                                        Choice (B)99. An aeroplane flying horizontally 1 km above the ground is observed by a person on his right side at an

elevation of 60°. If after 10 seconds the elevation is observed to be, from the same point and in the same direction, 30°, the uniform speed per hour (in km) of the aeroplane is (neglect the height of the person for computations)

      (A)                              (B)                                  (C) 720                            (D) 720 

Solution:     

      

Let P be the position of the man on the ground and B and C be the two positions of the aeroplane when it is observed.

      In  PAB, tan60° = 

       =    AP = 

Page 42: JMET 2006 Original Paper

      Now, in  PDC, tan30° = 

            + AD = 

       AD =   = 

 The speed of the aeroplane is 

      =    (60)  (60) km/hr = km/hr                                                                                                          Choice (B)

100. A certain sum of money is invested at an interest rate of 5% per annum and a second sum, twice as large as the first, is invested at 5.5% per annum. The total amount of interest earned from the two investments together is Rs. 1000 per year and the interest is withdrawn every year. The second sum invested is

       (A)  Rs.6250                         (B) Rs.10500                         (C) Rs.12500                         (D) Rs.15000

        Solution:       Let Rs.P be the first sum.       Then the second sum = Rs.2P       Given that

       P         0.05P + 0.11P = 1000        P = 6250        The second sum = 2P = Rs.12,500                                                                                                             Choice (C)101. P and Q start running in opposite directions (towards each other) on a circular track starting at

diametrically opposite points. They first meet after P has run for 75 meters and then they next meet after Q has run 100 meters after their first meeting. Assume that both of them are running at a constant speed. The length of the track (in meters) is

       (A)  70                                 (B) 175                                 (C) 250                                       (D) 350        Solution:       Let P and Q start simultaneously towards each other from A and B respectively.

Let they meet for the first time at M and the second time at N.        

Page 43: JMET 2006 Original Paper

       Given that   = 75 m and   = 100 mTo meet for the first time they together have to travel half the circumference and to meet for the second time they together have to travel a distance equal to the circumference.

       So, MB = =   = 50 m         AB = 75 + 50 = 125 m        The length of the circular track = 250 m                                                                                                       Choice (C)Questions 102 and 103 are based on the following

A bucket is in the shape of an inverted truncated right-circular cone with a base radius of 20 cm, and height 35 cm. The base angles, of a vertical cross section through the centre of the base, are 135° each. It contains

water whose height is 10 cm. A solid iron ball of radius 5 cm is dropped into the bucket.

 

102. The amount of water in the bucket (in cc) is

       (A)  7000                           (B) 8000                             (C) 19000  

                               (D) 27000

 

Solution:

 

 

      

 

 

      

      

 

 

       Volume of water =    

Page 44: JMET 2006 Original Paper

 

       Where h = height of water level = 10 cm

       R = Radius of water surface = 30 cm

       r = Radius of bottom = 10 cm

        Volume = 

       =                                                                                                                                                     Choice (C)

 

103. After the ball is dropped into the bucket, the height of the water in the bucket becomes

       (A)  35 cm                            (B) 30 cm                              (C) 25 cm                                   (D) 20 cm

 

       Solution:

       If we assume that the ball in totally submerged in the water, the volume of the displaced water in - equal to the volume of the ball.  

            

 

      

 

 

 

 

 

       Volume of displaced water = 

 

Where x = Radius of initial water surface = 30cm

Page 45: JMET 2006 Original Paper

       H = Change in water level

       x + y = Radius of sphere = 

          = 4(125) (74)

From the answer choices, the final levels could be 35, 30 , 25or 20 cm. Therefore, the change in water level could be 25, 20, 15 or 10 cm.

Substituting h = 10, we see that the equation is satisfied.

LHS = 10 [(30 + 10)2 + (30 + 10) (30) + 302] = 37000

       RHS = 37000

Therefore, the intended answer may have been 20cm. (The final level in initial level + change in level)

But this answer can’t be accepted as the assumption that the ball in totally submerged in water is not correct.

Diameter of ball = . It can’t be totally submerged. Actually, only the submerged part will displace water. Therefore the final level would be definitely less than 20 cm.

       Answer choice is not there.104. The function f(x) = mx + sin x will have an inverse if and only if

        (A)  – 1  m  1                     (B) m < – 1                       (C) m > 1                               (D)   > 1

 

       Solution:

       y = mx + sin x

       y' = m + cos x

As 1  cos x  1, if m = 1, 0  m + cos x  2

As y' is non-negative, the graph of the function does not go downwards anywhere and the function has an inverse.

If the graph goes upwards at places and downwards elsewhere, it would be a many to one function and the inverse would not exist.

       If m > 1, say m = 1 +  (where  > 0),

       as   cos x  1,   m + cos x  2 + 

       The inverse would exist.

       Similarly, if m  1, the inverse would exist.

Page 46: JMET 2006 Original Paper

 The given function would have an inverse if and only if |m|  1. But looking at the options given, the choice (D) seams to be most likely. So, we may go with it.                                                                                                                  Choice (D)

105. Ramesh has two examinations on Wednesday – Engineering Mathematics in the morning and Engineering Drawing in the afternoon. He has a fixed amount of time to read the textbooks of both these subjects on Tuesday. During this time he can read 80 pages of Engineering Mathematics and 100 pages of Engineering Drawing. Alternatively, he can also read 50 pages of Engineering Mathematics and 250 pages of Engineering Drawing. Assume that the amount of time it takes to read one page of the textbook of either subject is constant. Ramesh is confident about Engineering Drawing and wants to devote full time to reading Engineering Mathematics. The number of Engineering Mathematics text book pages he can read on Tuesday is

       (A)  500                               (B) 300                                 (C) 100                                 (D) 60        Solution:       Let us represent the given two subjects i.e. Engineering Mathematics and Engineering Drawing

by E.M. and E.D respectively.      It is given that, the total time with him is sufficient complete.

       80 pages of E.M. + 100 pages of E.D.--  (i)                            (or)       50 pages of E.M. + 250 pages of E.D.- - (ii)       Equating (i) and (ii)       80 pages of E.M. + 100 pages of E.D.       = 50 pages of E.M. + 250 pages of E.D.        30 pages of E.M. = 150 pages of E.D.        1 page of E.M. = 5 pages of E.D.      Now as he want to prepare only E.M., he can complete 80 pages of E.M., and in the time we can

complete 100 pages of E.D., he can complete 20 pages of E.M. i.e. a total of 100 pages.                                                                      Choice (C)

106.  equals        (A)  2                                     (B) 3                                       (C) 5                                       (D)         Solution:

       Let S = 

       S = 

       tn =   = 2

        S = 2

       = 2  = 2                                                                                                                                                   Choice (A)

107. A circular table is pushed to the corner of a room touching two perpendicular walls. If a point on the edge of the table facing the corner is 8 and 9 cm from the two walls then the radius of the table (in cm) is

        (A)  29                            (B) 17                              (C) 5                                     (D) undeterminable from above

        Solution:

Page 47: JMET 2006 Original Paper

               The point P on the edge of the table is 8 cm from one wall and 9 cm from the other       We see that       (R – 8)2 + (R – 9)2 = R2

                2R2 – 34R + 145 = R2

        (R – 5) ( R – 29) = 0        R = 29 (∵ R – 9 >

0)                                                                                                                                   Choice (A)108. When 4101 + 6101 is divided by 25, the remainder is        (A)  20                                   (B) 10                                     (C) 5                                      (D) 0        Solution:       Let S = 4101 + 6101

       = (5 – 1)101 + (5 + 1)101

= 2[101C0 (5)101 + 101C2 (5)99 + 101C4 (5)97 + ----+ 101C98 53 + 101C100 51]S = 2[25[101C0 599+100C2597+101C4595+----+101C985]  + 101(5)].

S = 2[25[K] + 505]S = 50K + 1010Clearly, 50K is exactly divisible by 25 and remainder obtained when 1010 is divided by 25 is 10. 40101 + 6101 divided by 25, remainder is 10.                                                                                                    Choice (B)

109. If   = 0, which of the following statements is TRUE?

        (A)  x, y, z are in HP or x,  , z are in AP

        (B)  x, y, z are in AP or x,  , z are in HP

        (C)  x,  , z are in HP or x, y, z are in GP

        (D)  x,  , z are in GP or x, y, z are in AP        Solution:       Given,

       

       

       

Page 48: JMET 2006 Original Paper

        ;        (2x – y) (3y – 32) = (2z – y) (x2 – xy)

2x3y – 2xz2 – zy2 + z2y = 2zx2 – 2zxy – yx2 + xy2

xy2 – x2y – 2xyz + zy2 –z2y  – 2zxy – 2zx2 + 2z2x = 0       [xy + zy – 2zx] [y – x – z] = 0       [y(x+z) – 2zx [y – x – z] = 0       y[x + z] – 2zx = or y – x – z = 0

       y =   or y = x + z

        x, y, z are in H.P or x,  , z are in A.P.                                                                                                       Choice (A)110. For non-zero real numbers a,b and c the set of possible values the quantity

    can take is  (A)  {0}                           (B) {– 4, 0, 4}                         (C) {– 4, –2, 2, 2, 4}               (D) {– 4, –2, 0, 2, 4}

        Solution:

       Let K =         The possible signs for a, b, c      

a b c K+ + + 4+ + – 0+ – + 0– + + 0+ – – 0– + – 0– – + 0– – – –4

 

  The possible values for the quantity  can take is {–4, 0, 4}                                               Choice (B)

111. In PQR,   AND mQPR = 20°. S is a point on PR such that mSQR = 60° and T is a point on PQ such that mTRQ = 50°. Then m STR equals

       (A)  60°    (B) 70° (C) 80° (D) 90° 

Page 49: JMET 2006 Original Paper

        Solution:                           

Given P = 20°, PQ = PR (i.e., Q = R = 80°)       SQR = 60°, TRQ = 50°

 TQS = 20° and SRT = 30°, QTR = 50° and hence QT = QR = x (say)Let TSQ =  and SQ = dIn STQ,

        ---   (1)And in SQR

             ---   (2)

  (1), (2)   

Page 50: JMET 2006 Original Paper

   sin (20° + ) = sin cos40°If the factor 2sin = 1 (i.e.,  = 30°) and 20° +  and 40° are complementary angles (i.e.,  = 30°) the equation is satisfied. i.e., by inspection we can write down the solution to this equation as  = 30°.

        STR = 80°.                                                                                                                                             Choice (C)112. The number of ordered triplets (x, y, z) such that x, y, z are primes and xy + 1 = z is        (A)  0                                    (B) 1                                      (C) 2                                     (D) infinitely

many        Solution:       Given,       xy + 1 = z

It is possible only for x = 2, y = 2 and z = 5. Other than this values no other triplets satisfy for x, y and z.

        The number of triplets is only one.                                                                                                               Choice (B)Questions 113 and 114 are based on the following:

 

Two men are walking towards each other alongside a railway track. A freight train overtakes one of them in 20 seconds and exactly 10 minutes later meets the other man coming from the opposite direction. The train passes this man in 18 seconds. Assume the velocities are constant throughout. 

113. How long after the train has passed the second man will the two men meet?

       (A)  89.7 minutes            (B) 90 minujtes                      (C) 90.3 minutes                    (D) cannot be determined from above

 

       Solution:

      

      

 

 

In 20 s, the train covers its own length (say L) with respect to the first man and in 18 s, it covers L with respect to the second man.

Let the speeds of the two men be a m/s and b m/s, and that of the train be V m/s.

          = 20 and   = 18

          = V – a and   = V + b

Page 51: JMET 2006 Original Paper

 

Subtracting the first equation from the second,

       b + a = L

 

In 20 s, the train covers L with respect to the first man. In the next 600 s (10 min), it covers 30L. Hence, it covers 31L with respect to the first man by the time it meets the second man.

This distance of 31L between the two men is covered in 31L/(a + b) or   min or 93 min OF there 93 min, the train takes 18 s to cross the second man.

As we are measuring time from the instant the train just crosses the second man, the time needed for the two men to meet is 93 min – 18 s = 92.7 min.                                                                                               Answer choice in not there.

 

114. The ratio of the velocities of the first man to the second man is

       (A)  – 10:9                      (B) 9:10                                (C) – 9:10                                   (D) undeterminable from above

 

       Solution:

       In the solution above

       We have

       V – a = L/20

       and V + b = L/18

       If we assume L = 180 k m

       V – a = 9k and V + b = 10k

        b = 10k – 9k + a = k + a

       where k could be anything.

       Thus we can’t determine b/a.                                                                                                                          Choice (D)

115. Laila drives to the station each day to pick up her husband Majnu, who usually arrives on a train at 6’O clock. Last Monday, Majnu finished work earlier, caught an earlier train and arrived at the station at 5 O' clock. He started to walk home and eventually met Laila who drove him the rest of the way, getting home 20 minutes earlier than usual. On Tuesday, he again finished early and found himself at the station at 5:30. Again he began to walk home, again he met Laila on the way, and she drove him home the rest of the way. Assume constant speed throughout with no wasted time for waiting, backing of the car etc. How much earlier than the usual time were they home on Tuesday?

Page 52: JMET 2006 Original Paper

        (A)  6 minutes                       (B) 8 minutes                          (C) 10 minutes                             (D) 12 minutes

        Solution:       Let us first of all talk the case on Monday.

On that day, Laila saved a total of 20 minutes or she took 20 minutes less than the normal day. She must have saved 10 minutes in journey to the station and 10 minutes in the return journey.The reason due to which she took 10 minutes less in the journey to the station is some of the distance is covered by Majnu by walk.Majnu usually met Laila at 6’O clock but on that day as Laila met Majnu 10 minutes before, they must have met at 5 : 50, i.e. which implies Majnu walked for a total of 50 minutes.The distance covered by Majnu is 50 minutes = the distance covered by Laila in 10 minutes.Now, on Tuesday, as the train arrived 30 min early, they must have met at 5 : 55, i.e.The distance covered by Majnu in 25 minutes = the distance covered by Laila in 5 minutes. Laila drove for 5 minutes less in the journey to the station, hence she saves a total of 10 minutes in the complete journey.                                                                                                                                                                    Choice (C)

116. What is the digit in the units place of 10251?        (A)  2                                   (B) 4                                     (C) 6                                     (D) 8             Solution:       Given,       S = (102)51

       We know that,       21 = 2, 22  = 4, 23 = 8, 24 = 16, 25 = 32 -------

For every successive four powers, unit’s digits repetition  follows the same cycle..       S = (102)51 = (102)45  (102)3

        (102)51 contains units digit as 8.                                                                                                                   Choice (D)

117. In PQR, m QPR = 45°, mPQR = 60° and  = 15  cm, QS is a median and T is the mid-

point of QS. PT extended meets QR at U. Then   equals        (A)  5 cm                             (B) 6 cm                                 (C) 7.5 cm                             (D) 10 cm        Solution:

                  

Let M be the foot of the perpendicular from R to PQ.

       PM = 

       MQ =        RQ = 2MQ = 15

Page 53: JMET 2006 Original Paper

                 

Let the line parallel to PU through S, intersect RQ at V. RV = VU =         QU = RQ/3 = 15/3 Cm = 5 cm.                                                                                                                    Choice (A)

118. The number of distinct real number x for which   is a positive integer is       (A)  3                                   (B) 4                                     (C) 5                                     (D) 8        Solution:       Let y = 4x – x2   = – (x2 – 4x) = – (x – 2)2 + 4        The graph of the function y = 4x –x2 is:

                            

Clearly, every real number between 0 and 4 is attainable by the function y = 4x – x2.For the values like 0.1, 0.01, 0.001 and so on for 4x – x2

         is a positive integer.

Hence, there are infinitely many values of x for which   is a positive integer. 

Page 54: JMET 2006 Original Paper

As we can see from the choices that none of them is true.

119. The minimum attainable value of the function f (x, y) =    is

  (A)  12                            (B) 13                              (C) 3 +                               (D) 4 +         Solution:

       Given f(x , y) = Above function contains two variables x and y.

       For maxima or minima,

        and   = 0

       

       

        

        

               Taking squares on both sides, we get       x2[x – y)2 + 4] = (x – y)2 (x2 + 1)       x2(x – y)2+4x2 = (x – y)2 x2 + (x – y)2

       4x2 = (x – y)2                  ---------- (1)

       

        = 0

               (x–y)2 [(12 – y)2 + 4) = (12 – y)2 (x – y)2 + 4]

(x – y)2 [(12 – y)2 + 4(x – y)2 = (12 – y)2 (x – y)2 + 4(12 – y)2 – (x – y)2 = (12 – y)2     ----- (2)       From (1) and (2), we get       4x2 = (12 – y)2

       [4x2 – (12 – y)2]  = 0       [2x + (12 – y] [2x – (12 – y)] = 0       (2x – y + 12) (2x + y –12) =0       2x – y + 12 = 0 or 2x – 12 + y = 0       y = 2x + 12  or y = –2x + 12 -------- (4)       Substituting (3) in (1), we get       4x2 = (x – 2x – 12)2

       4x2 = (–x –12)2

       2x = x + 12    or   2x = –x –12             x = 12     or        3x = –12             x = 12     or        x = –4       x = 12  y = 36

Page 55: JMET 2006 Original Paper

       x = –y  y       Substituting (4) in (1), we get       4x2 = (x + 2x – 12)2

       4x2 = (3x – 12)2

       2x = 3x – 12  or   2x = –3x + 12       –x = –12        or   5x = 12

       x = 12           or   x =        x = 12  y = –12

       x =    y =   By substituting the above values of x and y, we can observe that f(x, y) has minimum value

at 

  The minimum possible of f(x, y) is   = 13                                     Choice (B)

120. In a right-angled triangle PQR with  , M is a point on its hypotenuse PR. L and N  are feet of

the perpendiculars from M on PQ and QRrespectively.   will be minimized when        (A)  PQM and PQR are similar        (B)  M is the mid-point of PR        (C)  mPQM = mMQR = 45°

        (D)               Solution:

                          ML2 + MN2 = LN2 = QM2

Clearly QM has the minimum value when QM  PR, in which case  PQM ∼  PRQ and not PQRWhen we assert be the relation of similarity (or even congruence) between two triangles, the order in which the vertices are named is important.

        PQM   PRQ because        P of  PQM =  P OF  PRQ       And  Q of   PQM =  R of  PRQ

Strictly speaking, the correct answer is not there.But if a candidate ticks choice (A), he’ll probably be given credit for the question.                                               Choice (A)

SECTION 4: DATA INTERPRETATION (Questions 121 – 150)

 

Page 56: JMET 2006 Original Paper

Directions: Questions 121 – 126 are based on the following line graph displaying growth rates of exports of key manufacturing commodities from India:

 

 

 

 

 

 

 

 

 

 

 

 

 

 

 

 

 

 

 

Page 57: JMET 2006 Original Paper

121. Overall export growth for all commodities was the lowest during

       (A)  2001-02                         (B) 2002-03                            (C) 2003-04                           (D) 2004-05

 

       Solution:

       As the growth rate of all the commodities decreased in 2001-02, the overall export growth for all the commodities was the lowest during 2001-02.                                                                                                                                   Choice (A)

 

122. Which commodity reported rising growth rate of exports for two consecutive years?

       (A)  Engineering                    (B) Readymade Garments      (C) Textiles                           (D) Other

 

       Solution:

       Only 'Engineering' reported a rising growth rate of exports for two consecutive years i.e., 2002-03 and 2003-04.     Choice (A)

 

123. Which commodity has shown alternating (rising and falling) pattern of growth rate of exports from 2000-2001 to 2004-2005?

       (A)  Chemicals                     (B) Engineering                      (C) Textiles                           (D) Other

 

       Solution:

       Clearly 'Others' has shown alternating pattern of growth rate i.e., decrease and increase alternately.                    Choice (D)

 

124. Which commodity has shown the highest fluctuation in the growth rate of exports?

       (A)  Engineering                    (B) Chemicals                       (C) Textiles                           (D) Other

 

       Solution:

       Fluctuation is the highest for 'Engineering' between the years 2000-01 to 2001-02.                                              Choice (A)

 

125. Which commodities reported decline in the growth rate of exports for two consecutive years?

       (A)  Readymade Garments and Chemicals                          (B) Engineering and Textiles

Page 58: JMET 2006 Original Paper

       (C)  Textiles and Chemicals                                               (D) Textiles and Readymade Garments

 

       Solution:

       'Textiles' reported decline in 2003-04 and 2004-05. Similarly 'Readymade garments' reported decline in the same years.        

                                                                                                                                                                           Choice (D)

 

126. Which commodity has shown maximum improvement in the growth rate from 2000-2001 to 2004-2005?

       (A)  Readymade Garments    (B) Engineering                      (C) Textiles                           (D) Other

 

       Solution:

       The 'Others' reported growth from 5% to 30% i.e., 25 percentage points which is the highest.                              Choice (D)

Directions: Question 127 - 130 are based on the following radar chart displaying benchmarking scores on various parameters related to business excellence being pursued by a plant X of company Y for two years (2004 and 2005). For this purpose assume that the Company Y’s chart is the benchmark X is striving to achieve.

      

Page 59: JMET 2006 Original Paper

                127. On which of the two parameters there exists comparatively less gap between X-2004 and

Company Y?       (A)  People and Leadership          (B) People and Strategy    (C) Strategy and Leadership   (D) People and Customer Results        Solution:       From the chart, it can be clearly seen that X-2004 and company Y are close for 'People and

Leadership' i.e., 5 and 25 points respectively.                                                                                                                                                  Choice (A)

 128. Beyond 2005, to achieve the Company Y benchmark X should focus on       (A)  Customer Results                 (B) People                        (C) Leadership                       (D) Partnerships        Solution:       X should focus on the parameter which has less score than that of company Y, by a maximum

value. 'Partnerships' has the highest difference i.e., 30 points.                                                                                                                     Choice (D)

 129. The maximum improvement achieved by X in 2005 compared to 2004 was in       (A)  Customer Results                 (B) Strategy                     (C) Leadership                       (D) Partnerships        Solution:       The improvement is the maximum for 'Leadership' i.e., 60 – 25 = 35 points.

None among the others have more than 30 points.                                                                                            Choice (C) 

130. The only parameter in which X could reach the closest to the Company Y’s benchmark in 2004 was

       (A)  People                                 (B) Strategy                     (C) Customer Results                   (D) Processes        Solution:       In 2004, 'People' was the closest parameter for X with respect to company

Y.                                                     Choice (A)Directions: Questions 131 -136 are based on the following bar chart showing trends of the per capita net domestic product in some states and union territories (UTs) of India: 

Page 60: JMET 2006 Original Paper

                                                                        131. The average of values of per capital net domestic product for the states / UTs in 2002-03 lies

between Rs.       (A)  20,000-25,000                (B) 25,000-30,000                  (C) 30,000-35,000                  (D) 35,000-40,000        Solution:       The total per capita income (in '000s).

~ 18 + 47.5 + 25 + 20 + 24 + 25.5 + 37.5 + 20.5 = 218

Page 61: JMET 2006 Original Paper

       Average per capita income = ~ 27        27000 is between 25000 and 30000.                                                                                                             Choice (B) 132. For which state / UT was the per capita net domestic product farthest from the average of values

of per capita net domestic product of the states / UTs in 2002-03?       (A)  Chandigarh                    (B) A.P.                                (C) Kerala                                   (D) Pondicherry        Solution:       From the above problem, the average percapita income in 2002-03 was 27000 (approx).

The fartheset from this value should be either the highest or the lowest value.       For A.P, 27000 – 18000 = 9000       For Chandigarh, 47000 – 27000 = 20000.

The difference is the highest for Chandigarh.                                                                                                 Choice (A)

 133. Which of the following states / UTs has the highest growth rate in per capita net domestic

product between 2002-03 and 2004-05?       (A)  Kerala                           (B) Haryana                           (C) Chandigarh                            (D) A.P.        Solution:       The growth rates for all the states except Chandigarh, Haryana and Kerala are less than 20%.

       For Chandigarh, it is  100 ~ 21%

       For Haryana, it is  100 = 20%

       For Kerala, it is  100 = 25%        It is the highest for Kerala.                                                                                                                           Choice (A) 134. The number of states / UTs with more than Rs.2000 rise in per capita net domestic product in

any year over the previous year was       (A)  5                                   (B) 6                                     (C) 7                                                (D) 8        Solution       Except A.P. and Punjab, all other states had more than Rs.2000 rise in at least one of the given

years.       8 – 2 = 6                                                                                                                                                    Choice (B) 135. Which of the states / UTs had less than half the per capital net domestic product

of Chandigarh in each of the three years?       (A)  A.P., Kerala and Punjab                                              (B) A.P., Kerala and Tamilnadu       (C)  A.P., Kerala and Haryana                                            (D) A.P., Haryana and Tamilnadu        Solution:       Half of the per capita Net Domestic Product for Chandigarh in

       2002-03  = 23840

Page 62: JMET 2006 Original Paper

       2003-04   ~ 25670

       2004-05   ~ 28810 

 Only A.P., Kerala and Tamilnadu has less than these values in each of the given years.                                  Choice (B)

 136. The ratio of per capita net domestic product of a large developed (G7 member) Asian country

with per capita net product of US $40000, to per capita net domestic product of the richest state / UT among the given eight in 2004-05 (assuming US 1$= Rs.40) is in the range of

       (A)  15-20                             (B) 20-25                         (C) 25-30                                    (D) 30-35        Solution:       The value for the given country       = Rs.40000  40 = Rs.16  105

The highest value for any state UT is Rs.57,621.

       Ratio ~~ 27                                                                                                                                  Choice (C)Directions: Questions 137 - 144 are based on the following tables and line graph about GNP and trade for select countries:

 Trends in National Product and Trade for Large countries

GNP per Capita in US $Countries 2000 2001 2002 2003 2004

China 930 1,000 1,100 1,270 1,500Germany 25,510 24,000 23,030 25,700 25,500

India 450 460 470 540 620Japan 35,140 35,670 33,640 33,860 37,050U.S.A 34,400 34,800 35,230 37,780 41,400

 Merchandise Trade as % of GNP

  2000 2001 2002 2003 2004China 40 38 43 52 60

Germany 55 56 55 56 59India 21 20 21 21 25Japan 18 18 19 20 20U.S.A 21 19 18 19 20

 

Page 63: JMET 2006 Original Paper

            

          137. In which year did all countries show growth in terms of increase in GNP per capita compared to

the previous year?       (A)  2001                        (B) 2002                                (C) 2003                                (D) 2004        Solution:

The GNP grew for all countries in only 2003.                                                                                                    Choice (C)

 138. Which of the following two countries have shown consistent growth in GNP per capita during 2000 to

2004?       (A)  China and Germany  (B) China and Japan               (C) Germany and Japan          (D) India and U.S.A        Solution:       Out of the given countries, only China, India and U.S.A. has shown growth in each and every

year.                      Choice (D) 139. What was the approximate percentage increase in GNP per capita from 2000 to 2004 for the

country that achieved the maximum percentage jump in GNP per capita in any year?        Solution:       No country had more than 10% increase (in any year) except China and India.

       For China, the highest increase rate =  100 ~ 18

Page 64: JMET 2006 Original Paper

       For India, it is  100 ~ 15

For China, the percentage increase from 2000 to 2004 = 100  ~ 61.                                                       Choice (C)

 140. What was the approximate ratio is 2004 of total GNP for India to the company with the lowest

average merchandize trade as percentage of GNP, if ratio of population between the two countries in 2004 was 10:1?

       (A)  1 : 2                              (B) 1 : 4                                (C) 1 : 6                                (D) 1 : 8        Solution:       Japan has comparatively a least values in all the years for Merchandise trade as % of GNP.       The total GNP for it in 2004 = 37050       And total GNP for India = 10  620 = 6200       Ratio of India to China = 6200 : 37050       ~ 1 : 6                                                                                                                                                          Choice (C) 141. If India’s GNP was about Rs.28,00,000 crore in 2004, what was approximate value of India’s

merchandize trade in US 1$ billion in the same year (assume US 1$ = Rs.50)?       (A)  130                               (B) 140                                 (C) 150                                 (D) 160        Solution:

       India's GNP = crores = $56,000 crores

       India Merchandise trade in 2004 =  56000       = $14000 crores = $140 billion.                                                                                                                       Choice (B) 142. What was the ratio of exports to imports in merchandize trade for India in 2004, if exports were

about US $60 billion, given that India’s GNP was about Rs. 28,000,000 crore? (assume US 1$ = Rs. 50)

       (A)  1 : 2                              (B) 3 : 4                                (C) 5 : 6                                (D) 1 : 1        Solution:

From the above question, India's Merchandise trade is $140 billion.       Exports = $60 billion        Imports = 140 – 60 = $80 billion.       Ratio = 60 : 80 = 3 : 4                                                                                                                                 Choice (B) 143. What was the approximate difference in dollar value of high-tech exports

between China and India in 2004, if their exports of merchandize products were US $ 500 billion and US $ 60 billion respectively?

       (A)  130                               (B) 137                                 (C) 140                                 (D) 145        Solution:       High-tech exports for China in 2004

       =  500 = $150 billion

       Similarly for India it is   60 = $3 billion 

Page 65: JMET 2006 Original Paper

       Difference = $147 billion.                                                                                                                             Choice (D) 144. Which of the following is TRUE?       (A)  U.S.A. has consistently shown decline in the merchandise trade as percentage of GNP       (B)  Japan has consistently shown growth in GNP per capita       (C)  Germany has consistently shown growth in high-tech exports as percentage of merchandise

exports       (D)  China has consistently shown growth in high-tech exports as % of merchandise trade and GNP

per capita        Solution:       Choice (A) is false as there was an increase from 2002 onwards.

Choice (B) is false as there was decrease from 2001 to 2002.Choice (C) is false as there was decrease from 2001 through 2003.

       Choice (D) is true.                                                                                                                                         Choice (D)

Directions: Questions 145-150 are based on the following bar chart of trends in sales of motorcycles in India and the table of Trends in sales for Bajaj:

                                                                                                                             

    

  

 Trends in Sales (Rs. In crores) for Bajaj

 

Product 2000-01 2001-02 2002-03 2003-04Motorcycles 1277 1893 2608 3185Scooters 998 1037 750 527Three Wheelers 880 975 1154 1443

 

145. Which company had experienced minimum percentage growth rate in motorcycle sales from 2001-02 to 2003-04?

Page 66: JMET 2006 Original Paper

       (A)  Baja                              (B) LML                                (C) TVS                                (D) Hero Honda        Solution:       For LML, TVS and Bajaj, the sales of motor cycles had become more than twice.       For Hero Honda, percentage growth

=  100 ~ 83%, which is the least.                                                                                              Choice (D)

 146. In which year, the percentage growth compared to the pervious year in total sales for motorcycles

was the maximum?       (A)  2000-01                         (B) 2001-02                           (C) 2002-03                           (D) 2003-04        Solution:       For 2001-02, the percentage growth

         100 ~ 36%

       For 2002-03, it is  100       = 33%       For 2003-04, it is 10%       It is the highest for 2001-02.                                                                                                                         Choice (B) 147. What was the approximate ratio of growth of sales of motorcycles for Hero Honda to that of Bajaj

from the year 2001-02 to 2002-03?       (A)  1 : 2                              (B) 6 : 7                                (C) 5 : 4                                (D) 7 : 6        Solution:       For Hero Honda, the growth of sales = 4700 – 4200 = 500       Similarly, for Bajaj, it is 2608 – 1893 = 715       Ratio = 500 : 715 ~ 2 : 3                                                                                                             Choice (Not in options) 148. What was the approximate share of three wheelers in the three products manufactured by Bajaj in

the year during which the company experienced maximum rise in the sales of motor cycles compared to the pervious year?

       (A)  40%                              (B) 35%                                (C) 26%                                (D) 20%        Solution:       The maximum rise in sales of motorcycles for Bajaj is in 2002-03.

       Share of three wheelers in 2002-03 =  100  = 100 ~ 25.6                                       Choice (C) 149. For Bajaj, in which year, the motorcycle sales overtook the combined sales of scooters and three

wheelers?       (A)  2001-02                         (B) 2002-03                           (C) 2003-04                           (D) None        Solution:       In 2001-02, 975 + 1037 > 1893       In 2002-03, 1154 + 750 < 2608

The motorcycle sales overtook the combined sales of others in 2002-03.                                                         Choice (B)

 150. If motorcycle, scooter and three wheeler markets grew by 20, 0 and 40% respectively in 2004-05

compared to 2003-04 uniformly for all players, what was the approximate ratio of total sales of Hero

Page 67: JMET 2006 Original Paper

Honda to Bajaj in 2004-05? (assume that Hero Honda does not manufacture scooters and three wheelers)

       (A)  2 : 1                              (B) 1.5 : 1                             (C) 1 : 1                                (D)   1 : 1.5        Solution:

       Total sales of Hero Honda in 2004-05 =  5500 = 6600

       Total sales of Bajaj in 2004-05 =  = 3822 + 527 + 2020 = 6369       Ratio = 6600 : 6369 ~ 1 : 1                                                                                                                          Choice (C)